Download table of contents - Northwestern Medical Review

Document related concepts

Medical ethics wikipedia , lookup

List of medical mnemonics wikipedia , lookup

Transcript
TABLE OF CONTENTS
Topics/Chapters
Page Location
Nephritic Syndrome and Membranoproliferative
Glomerulonephritis (MPGN): Renal Pathophysiology
Staph aureus: Integrated Bacteriology Review
1
Adrenal Pathophysiology and Pharmacology:
Integrated Endocrinology
Aortic Stenosis: Ventricular Pressure-Volume Loop
Pathophysiology
Visual Field Defects: Neurology and Mini
Neuroanatomy
Glycolysis and Glycolytic Enzyme Deficiencies:
Biochemistry
Simulated Post-test
14
9
23
28
37
48
Nephritic Syndrome and
Membranoproliferative Glomerulonephritis
Excerpts of Renal Pathophysiology
Synopsis of Nephritic Syndrome and Membranoproliferative Glomerulonephritis (MPGN)
Your patient is an 18 year-old woman who is seen for
the complaint of occasional vomiting, back pain,
swollen ankles, and oliguria. She has a 4-year history of
arthritic joint pain. She previously tested positive for
serum antinuclear antibody (ANA). On examination she
has a blood pressure of 160/90 mmHg. Urinalysis is
significant for hematuria, and serology shows high BUN
and creatinine levels. To confirm your clinical suspicion,
you schedule her for renal biopsy and
immunofluorescence evaluation. Results of the biopsy
show a tram-track appearance of the glomerular
basement membrane and sub-endothelial deposits of
immune complexes.
1. It is apparent that the patient has renal disease.
Which of the two patterns of renal disease,
nephrotic or nephritic, is supported by the given
findings?
_______________________________________
_______________________________________
2. What is the classic triad of nephritic syndrome?
_______________________________________
_______________________________________
6. What are the major causes of nephritic syndrome?
P____________________________________
I ____________________________________
G____________________________________
Rapidly progresses__________________
to
H____________________________________
A____________________________________
M____________________________________
Hematuria!
PIG Rapidly Progresses to HAM!
"If slaughterhouses had glass walls,
everyone would be vegetarian.”
Sir Paul McCartney
PETA Organization, UK
*Imagine that the number “2” is in superscript to
remind you of HYPER-tension!
3. What is the mechanism of severe hypertension and
oliguria in nephritic syndrome?
_______________________________________
_______________________________________
_______________________________________
_______________________________________
4. Hematuria of nephritic syndrome is known as
glomerular hematuria. How do we distinguish
between hematuria of glomerular origin and those
that originate from lower urinary tract locations?
_______________________________________
_______________________________________
5. What is the mechanism of red cell cast formation in
nephritic syndrome?
________________________________________
________________________________________
Question 7 Refers to the previous clinical vignette:
7. Which of the listed conditions that we covered with
the mnemonic of “PIG RAPIDLY PROGRESSes to
HAM” is the most likely cause of her symptoms?
________________________________________
________________________________________
Hint: the tram-track appearance of the glomerular basement
membrane and subendothelial deposits of immune complexes!
Supplemental Drill Questions
8. What is the other commonly used term for rapidly
progressive glomerulonephritis?
________________________________________
________________________________________
9. What is the hallmark of nephritic syndrome?
________________________________________
________________________________________
1. Northwestern Medical Review, Bare Minimum Pathology Review Series, 2012
10. What is the hallmark of nephrotic syndrome?
________________________________________
________________________________________

11. It is apparent that the procedure for collecting 24hour urine samples for measuring protein levels is
quite difficult for patients as well as being more
time-consuming and expensive. What is the
commonly used alternative test for this purpose?
________________________________________
________________________________________

12. Single dipstick (spot) urinalysis tests are reported
in four grades of 1+, 2+, 3+ and 4+. What does this
reporting mean and what is the reason that
proteinuria of more than 3.5 is known as nephrotic
range proteinuria?
_______________________________________
_______________________________________

13. A patient has a single spot urine test of 2+; what
does this mean?
________________________________________
________________________________________



MPGN
MPGN is an uncommon cause of nephritic
syndrome that mostly affects children and young
adults.
There are two major types of MPGN, mesangiocapillary glomerulonephritis (known as Type I), and
mesangial proliferative glomerulonephritis (known
as Type II).
Causes: Idiopathic or familial predispositions.
Type I MPGN is due to abnormal immune
response and sub-endothelial antibody and
immune complex deposition.
In either Type I or II, mesangial cells increase in
number.
Type I, as its name implies, is by far the most
common and most commonly tested one on the
USMLE and COMLEX examinations.
Note: Type III MPGN is uncommon and is characterized by
the concomitant presence of sub-endothelial and
subepithelial deposits.
Type I MPGN
Type II MPGN
 AKA. Mesangiocapillary proliferative
glomerulonephritis
 The more common type
 Immune complexes activate the classical
complement pathway
 Subendothelial and mesangial deposits
 Thickening of the capillary walls and interposition
of mesangial cytoplasm into the peripheral
capillary loops
 Mesangial cell proliferation
 AKA. Mesangial proliferative glomerulonephritis
 No immune complexes involved
 Pathogenesis is related to uncontrolled activation of
the alternate complement pathway
 Dense homogenous deposition along the
glomerular basement membrane and in the
mesangium
 Mesangial cell proliferation
 Dense deposits largely diminish kidney’s filtering
ability
14. What is the function of mesangial cells?
_______________________________________
_______________________________________
15. What does the following
diagram remind you of and
how is it related to nephritic
syndrome in general and
MPGN in particular?
_______________________________________
_______________________________________
Hint: Supreme pizza with lots of items!
16. Would you be able to identify hypercellularity of the
glomeruli in light microscopy of other causes of
nephritic syndrome besides poststreptococcal
glomerulonephritis?
________________________________________
________________________________________
17. What is the basement membrane?
________________________________________
________________________________________
18. What is basal lamina and how does it differ from
the basement membrane?
________________________________________
________________________________________
2. Northwestern Medical Review, Bare Minimum Pathology Review Series, 2012
19. What is the location and function of the basal
lamina?
_______________________________________
_______________________________________
20. What are the 3 layers of the basal lamina and how
do they help to differentiate the basement
membrane from the basal lamina?
_______________________________________
_______________________________________
21. What are the major chemical ingredients of the
laminal layers?
________________________________________
________________________________________
22. What staining technique is most helpful in
highlighting the basement membrane for
visualization in light microscopy because of the
presence of glycoproteins (e.g. heparan sulfate)?
________________________________________
________________________________________
Hint: The color scheme of the above picture must have been a
pink-red color!
23. An 18 year-old patient with a recent history of
streptococcal sore throat presents with sudden
onset of hematuria and hypertension. Within 4
days his condition worsened and his 24-hour urine
protein exceeded 3.5 grams. A renal biopsy was
performed. Results were significant for crescentic
urinary spaces. What is your most probable
diagnosis?
_______________________________________
Type I MPGN characteristic microscopic findings are
increased number of mesangial cells, thickened
basement membrane, extension of mesangium in
between the epithelial and endothelial cells, narrow
lumen of the vessels, and widespread mesangial and
subendothelial deposits of immune complexes.
Type II MPGN
Type II MPGN presents with proliferation of mesangial
cells, lack of subendothelial deposits, and thickened
basement membrane (see the above diagram). The
thickened basement membrane may be due to
combined growth of mesangial matrix and basement
membrane.
24. What are the two major microscopic findings of
MPGN 1?
________________________________________
________________________________________
25. You might have heard of the term “tram track” in
conjunction with MPGN; what does it really mean?
________________________________________
________________________________________
Type I MPGN
The answer will be evident soon!
*The dumbbell-shaped structure in the left capillary is a red blood cell!
26. What other staining method will highlight the
basement membrane?
_______________________________________
3. Northwestern Medical Review, Bare Minimum Pathology Review Series, 2012
31. Does nephritic or nephrotic pattern of MPGN
present in a rapidly or slowly progressive manner?
________________________________________
________________________________________
32. Which of the two MPGNs is called dense deposit
disease?
________________________________________
________________________________________
Back to our original case-scenario…
Your patient is an 18 year-old woman who is seen for the complaint
of occasional vomiting, back pain, swollen ankles, and oliguria. She
has a 4-year history of arthritic joint pain. She previously tested
positive for serum antinuclear antibody (ANA). On examination she
has a blood pressure of 160/90 mmHg. Urinalysis is significant for
hematuria, and serology shows high BUN and creatinine levels. To
confirm your clinical suspicion, you schedule her for renal biopsy
and immunofluorescence evaluation. Results of the biopsy show a
tram-track appearance of the glomerular basement membrane and
sub-endothelial deposits of immune complexes.
27. Which of the following serological findings would
you also expect in this patient?
A. Low levels of serum complement 3 (C3)
B. High levels of serum complement 3 (C3)
C. High levels of IgG auto-antibody against C3
convertase that over-activates the alternative
complement pathway
D. Low levels of C1-esterase enzyme that overactivates the classic pathway
E. Low levels of serum complement 2 (C2)
28. Formerly we said that it is postulated that
complement deficiency plays an important role in
the pathogenesis of MPGN. What complement
deficiency is most commonly involved in this
situation?
________________________________________
________________________________________
29. What is the prognosis of MPGN?
________________________________________
________________________________________
33. Although the tram-track appearance is characteristic of MPGN, it is not as apparent in one of the
two types; in which type is it less apparent?
________________________________________
________________________________________
34. Why do we only see the tram-track appearance in
the above type of MPGN?
________________________________________
________________________________________
35. The case scenario states that the patient had been
tested positive for antinuclear antibodies; what is
the significance of this piece of information?
________________________________________
________________________________________
36. Name important autoimmune conditions that are
associated with ANA.
________________________________________
________________________________________
37. The gender and age of the patient may increase
the suspicion of SLE as our top differential for the
ANA antibodies. Assuming that the patient has
SLE, is SLE nephropathy a nephrotic or nephritic
syndrome?
________________________________________
________________________________________
38. IgA nephropathy is a non-systemic disease that is
associated with mesangial IgA deposits. But
several systemic diseases are also associated with
mesangial IgA deposits. Name the top 4 such
diseases.
________________________________________
________________________________________
________________________________________
30. Which of the two patterns of nephritic or nephrotic
syndrome would you expect to see in MPGN?
________________________________________
________________________________________
4. Northwestern Medical Review, Bare Minimum Pathology Review Series, 2012
39. Patients with ankylosing spondylitis are also
presented with mesangial deposits. These patients
are often young males that, in addition to IgA
deposits, may also have another type of immune
deposit in their glomeruli and vertebral joints. What
is the name of the other immune deposits, and why
is this condition more common in young males?
________________________________________
________________________________________
40. Dermatitis herpetiformis is also associated with IgA
nephropathy. What is the relationship between
herpes and dermatitis herpetiformis?
________________________________________
________________________________________
41. What type of deposits would you expect to see in
the glomeruli of a patient with SLE nephropathy?
________________________________________
________________________________________
42. A patient with a history of systemic lupus
erythematosus presents with MPGN and
glomerular IgA deposits. Which of the two patterns
of MPGN would you expect in this patient: Type I
or II?
________________________________________
________________________________________
Simplified Classic and Alternate Complement Pathways








Details of the Classic Complement Pathway
The classic pathway shown above is a simplified
an oldest version of the classic pathway. The
current view of the classic pathway includes the
following steps:
The classic pathway starts with an antigenantibody reaction. The antibody is either IgG or
IgM.
Complement 1 (C1) identifies Fc receptors of the
IgG or IgM
C1 is activated and forms C1qrs complex. Note
that C1 is a multi-subunit protein containing three
different proteins (C1q, C1r and C1s)
C1qrs activates C4
Activated C4 breaks down to C4a and C4b
C4b activates C2 to form C4b2a complex
C4b2a complex is also known as C3 convertase





C4b2a activates C3 (note that calcium is necessary
for this reaction)
Activated C3 breaks down to C3a and C3b
fragments
C3b binds to the membrane in association with
C4b and C2a. Meanwhile C3a is released into the
surrounding area. The resulting C4bC2aC3b is
known as C5 convertase. Generation of C5
convertase is the end of the classic pathway.
C5 convertase activates MAC complex.
Activated MAC has lytic power and attacks cellular
membranes of pathogens, makes pores in them,
and mediates both pathogenesis and the
prevention of immune complex diseases.
5. Northwestern Medical Review, Bare Minimum Pathology Review Series, 2012












Details of the Alternate Complement Pathway
C3 is made by the liver and is abundant in the
blood.
The
pathway
is
initiated
by
the
spontaneous hydrolysis of C3, that is, C3 (H2O)
formation.
Binding of plasma protein Factor B to hydroxylated
C3 allows Factor D to cleave Factor B into Ba and
Bb fragments.
Bb attaches to C3 to form C3Bb (AKA. C3
convertase). The latter cleaves many C3
complements into C3a and C3b.
C3a dissipates and C3b cleaves C5 to C5a and
C5b. C5a dissipates.
C5b, C6, C7, and C9 together form the MAC
complex.
Lectin (Mannose-Binding Lectin) Pathway
The lectin pathway is the third complement
pathway.
Mannose-binding protein is produced by
the liver and
can
initiate
the complement
cascade by binding to pathogen surfaces.
It binds to mannose, glucose, or other sugars or
glycoprotein components of micro-organisms.
It is similar to the classical complement pathway
because after activation, it proceeds through the
action of C4 and C2 to produce activated
complement proteins further down the cascade.
In contrast to the classical complement pathway, it
does not recognize antibodies bound to its target.
The pathway starts with mannose-binding
lectin binding to certain sugars on microorganisms.
43. What is the consequence of activation of MAC
complex in the nephrons?
________________________________________
________________________________________
46. Which of the two immunoglobulins, IgG or IgM, is
more effective in binding complements and
initiating the complement cascade?
________________________________________
________________________________________
47. Biopsy of the glomeruli and immunofluorescence
studies of the deposits in renal diseases show
certain patterns of distribution of immune
complexes. Which of the following patterns is
associated with mesangial deposits?
________________________________________
________________________________________
Fireworks in the Sky!
48. MPGN type II at times is associated with acquired
partial lipodystrophy (APL). Patients present with
whitish-yellow deposits within Bruch's membrane
of the retina quite commonly during their second
decade of life. Over time these deposits may cause
macular detachment, and central serous
retinopathy. What is the term that is used to
describe those whitish-yellow deposits?
________________________________________
Note: Bruch's membrane is the inner layer of the choroid, separating it
from the pigmentary layer of the retina.
44. Activation of which of the two complement
pathways, classic or alternate, is most likely related
to etiology of IgA nephropathy and Type II MPGN?
________________________________________
________________________________________
45. Which of the two complement systems, classic or
alternate, is more effective against the lipopolysaccharide of gram-negative bugs?
________________________________________
________________________________________
6. Northwestern Medical Review, Bare Minimum Pathology Review Series, 2012
Answers
1.
Nephritic
2.
Hypertension, Hematuria, and Oliguria
3.
Immune complex injury of the glomerular capillary
endothelium causes hematuria. Meanwhile, widespread
damaged glomerular vessel walls will activate
degranulation of the platelets and lead to thrombotic
blockade of the vessels and a resulting drop in the GFR
and oliguria. Drop in the GFR will aggressively activate
the renin-angiotensin system, and this leads to
hypertension.
4.
Hematuria of glomerular origin presents with red blood
cell casts, whereas the hematuria that originates after
the renal pelvis does not present with casts.
5.
Red cells that pass through injured fenestrations of the
glomerular capillaries are damaged and assume
dysmorphic shapes. Additionally, they pile up and
adhere to each other with protein molecules. As such
they assume a stack-like and cylindrical appearance.
6.
“PIG RAPIDLY PROGRESSes to HAM!” Stands for
Poststreptococcal Glomerulonephritis, IgA nephropathy,
Goodpasture’s syndrome, Rapidly Progressive
glomerulonephritis, Henoch-Schonlein purpura, Alport’s
syndrome, and Membranoproliferative glomerulonephritis.
7.
Membranoproliferative glomerulonephritis.
8.
Rapidly progressive glomerulonephritis is also known as
crescentic nephritis.
9.
Hematuria
will be lost. Note that even if a patient loses 4, 5 or more
grams of protein per day it is going to be reported as 4+.
13. Pre-nephrotic range or 2 grams per day of protein loss.
Note: Although the lab reporting may vary from lab to
lab, below is a helpful summary table based on the
recommendations of the International Society on
Nephrology.
Grades
I
II
III
IV
Description
1+ on dipstick or 0.15–1.0 g/24 2+ to 3+ on dipstick or >1.0–3.5 g/24 h
4+ on dipstick or >3.5 g/24 h
Nephrotic syndrome
14. Mesangial cells, modified smooth muscle cells, occupy a
central position within the renal glomerulus. They
produce cytokines and prostaglandins, mediate
inflammation, produce and remove basement
membrane and other matrix substances, uptake immune
complexes, and by the virtue of their contraction or
relaxation-ability can modify glomerular filtration rate.
15. This diagram is a cartoon version of hypercellularity of
the glomeruli in poststreptococcal glomerulonephritis
(PSGN). The cells (and their nuclei) are practically all
types of cells; endothelial, mesangial, epithelial,
neutrophils, etc.
16. Hypercellularity is a common finding of nephritic
syndrome, although it may not present with a diffuse
pattern as it is seen with PSGN. For instance, in Type I
MPGN it is more localized to the mesangial cells.
17. Extracellular matrix found on the basal surface of and
secreted by the epithelial cells.
10. Proteinuria of more than 3.5 grams per day
11. Single “Spot” urine test (simple dipstick test). Nephrotic
range proteinuria is 3 grams per day or more.
12. On a single, "spot" urine collection, it is 2 grams of
protein (albumin) per gram of urine creatinine. A healthy
liver makes 3 grams of proteins (albumin) per day. If in
nephrotic syndrome a patient loses 1 (1+), 2 (2+), or 3
(3+) grams of albumin per day, the liver compensates by
making up to 3 grams per day. However, above 3 (4+)
grams of protein loss, the liver’s compensatory ability
18. The basal lamina and the basement membrane are
quite often used interchangeably. However, the term
"basal lamina" is usually used in conjunction with
electron microscopy, while "basement membrane" is
usually used with light microscopy.
19. The basal lamina surrounds and lies underneath sheets
of epithelial cells. In the lungs and kidneys it separates
two types of cells; namely, the endothelial cells of blood
vessels and epithelial cells from each other.
20. The 3 layers of basal lamina are lucida, densa, and
reticularis. Some histologists believe that the basal
lamina is composed of lamina densa and lamina lucida,
whereas the basement membrane is composed of the
lamina densa and lamina reticularis. No matter which
description of basement membrane and basal lamina
you subscribe to, they both have the middle layer,
lamina densa, in common!
7. Northwestern Medical Review, Bare Minimum Pathology Review Series, 2012
21. Collagen Type IV
glycosaminoglycan)
and
Heparan
sulfate
(a
22. Periodic Acid Schiff (PAS)
23. Rapidly progressive glomerulonephritis
24. Mesangial and subendothelial deposits of immune
complexes, and increased number of mesangial cells.
25. Tram-track refers to the basement membrane in MPGN,
which has a tram-track appearance in light microscopy
with the help of special staining techniques. With PAS
staining, the capillary endothelial lamina and basement
membrane show two distinct membranes in light
microscopy due to mesangial interposition into the
capillary, which gives the loops a tram-track
appearance. Do all MPGN types have tram-tracks?
Please see below for the answer.
26. Silver staining (methenamine-silver staining) is the
second commonly used staining, besides PAS, for
highlighting the basement membrane. With this type of
staining the glomeruli will assume a silverish-bluish
coloration.
27. [A]. Low levels of serum complement 3
28. C3 deficiency is very common in MPGN and is
demonstrable in about 75% of the patients.
29. As you might have guessed the prognosis of MPGN is
relatively poor despite optimal effects of steroids and
immune suppressant therapy. The disease often
progresses to end stage renal failure.
30. The best answer is either, but the nephritic pattern is by
far the most common pattern. Quite often it starts with a
nephritic pattern and progresses to a nephrotic pattern.
31. They both appear in a slowly progressive pattern. This is
their distinguishing characteristic against rapidly
progressive glomerulonephritis.
32. Type II
33. You don’t see it in Type II and you only see it in Type I
MPGN.
34. In Type I the mesangium extends between the podocytes and endothelial cells, whereas in Type II we get a
dense homogenous deposition along the glomerular
basement membrane and in the mesangium that would
not allow the double contour to express itself. In Type II
you will see a thick ribbon-like pattern.
35. This supports the fact that MPGN must have an
autoimmune etiology!
36. SLE, Sjogren’s, Scleroderma, Polymyositis and
Rheumatoid arthritis.
37. SLE presents with either nephrotic or nephritic or a
mixed pattern. But the nephrotic pattern is by far the
most commonly presented pattern.
38. Henoch-Schonlein Purpura, Systemic lupus erythematosus, Ankylosing spondylitis, and Dermatitis
herpetiformis. A few other noteworthy associations are
celiac disease, rheumatoid arthritis, and HIV.
39. Ankylosing spondylitis is due to IgA and more
importantly, HLA-B27 deposits. This condition is
characteristically more common in men, and is
accentuated with hyper-testosterone levels in younger
men. It is postulated that testosterone facilitates
interaction of HLA-B27 with the glomerular or joint
tissues.
40. There is no relationship between the two conditions. The
presumption of a relationship may be due to the
resemblance of the skin rash of dermatitis herpetiformis
with that of the rash of herpes. Interestingly, dermatitis
herpetiformis is heavily associated with celiac disease
and gluten sensitivity.
41. SLE nephropathy may have any combination of IgG,
IgM, IgA, and C3 glomerular deposits.
42. The patient with SLE will most likely have a Type II
pattern of MPGN. In Type I, immune complexes are
due to IgG and IgM, and they activate the classical
complement pathway. In contrast, Type II MPGN
involves uncontrolled activation of the alternate
complement pathway and consumption of C3. Hence,
this patient most likely has Type II MPGN due to IgA
deposits.
43. Ongoing activation of the complements and MAC
complex in the nephrons in nephritic and nephrotic
diseases can cause ongoing damage to the kidney’s
filtering ability.
44. Given that IgG and IgM are involved in the initiation of
the classic pathway, the IgA nephropathy and MPGN
Type II must involve the alternate complement pathway.
45. The alternate pathway.
46. IgM is more effective because it has 5 immunoglobulin
subunits and thus more binding surfaces.
47. [C]. This pattern resembles fireworks in the sky, or “4th
of July pattern!”
48. Drusen’s deposits
8. Northwestern Medical Review, Bare Minimum Pathology Review Series, 2012
Excerpts
Bare Minimum Microbiology Review
Staph aureus
9. Northwestern Medical Review, Bare Minimum Microbiology, 2012
Staphylococcus aureus
 Gram-positive cocci, grape-like clusters, facultative anaerobic, catalase and coagulase-positive,
and grows on blood agar.
 Reservoir: Skin and nose;
 Transmission: By hands is an important issue
 Important diseases
 Abscess: Protein A, coagulase, leukocidin
 Gastroenteritis (food poisoning): Enterotoxin
 Scalded skin syndrome: Exfoliatin
 Toxic shock syndrome: TSST
 Acute bacterial endocarditis
 Hospital Acquired pneumonia
 Others: Various types of skin infections; pimples, impetigo, boils, cellulitis, carbuncles, etc.,
meningitis, osteomyelitis, and septicemia.
 Important Issue:
 Common cause of nosocomial infections
 Antibiotic Resistance: (1) Beta-lactamase (Penicillinase); and (2) Altered binding site for
Penicillin (MRSA)
 Treatment: Nafcillin (for all but MRSA), and Vancomycin for MRSA
Side Effects of Vancomycin
Thrombophlebitis
Ototoxicity
Nephrotoxicity
Shock
Van has TONs of Load!
1. What is the mechanism of action of penicillin?
_____________________________________
2. What is the other name for beta-lactamase?
_____________________________________
3. What is the mechanism of action of
Vancomycin?
_____________________________________
_____________________________________
4. What is the bactericidal spectrum of
Vancomycin?
Gram _______________________________!
Hint: What sign do you see on the van?
5. What is the next DOC for MRSA?
_____________________________________
Indications of Vancomycin
Methicillin Resistant Staph aureus
Pseudomembraneous colitis
Prosthetic heart valve patients who undergo
oral surgery (Staph epidermidis)
* May also be used for serious cases of resistant
streptococcus pneumoniae.
10. Northwestern Medical Review, Bare Minimum Microbiology, 2012
6. Staphylococcus and streptococcus cultures
produce either a grape-like cluster or have a
chain-like appearance. Which one is which?
____________________________________
12. What is scombroid and what are the major
sources of poisoning with it?
_____________________________________
_____________________________________
7. Staph aureus is the most common cause of
abscess formation. What are the characteristic
immune cells within an abscess?
____________________________________
13. How long is the incubation period of Staph
aureus food poisoning?
___________________________________
8. Of the 3 virulent mechanisms causing abscess
formation by staph aureus, one is protein A
which coats staph with immunoglobulin; and
the second one is coagulase that converts
fibrinogen to fibrin. What is the function of the
third one, leukocidin?
_____________________________________
9. The key organisms that one must differentiate
from Staph are Strep and Listeria. Is Strep
catalase-positive or negative?
_____________________________________
10. Is Listeria catalase-positive or negative?
_____________________________________
14. How long is the incubation period of
Salmonella food poisoning?
___________________________________
15. S. aureus, among other things, is one the two
must-know causes of abscess formation. What
is the other important abscess producer?
_____________________________________
Hint 1: Anaerobic
Hint 2: Gram-negative rod
Hint 3: Bowel surgery
Hint 4: Metronidazole and Clindamycin
16. There are only 5 must-know anaerobic bugs
for the exam; one of the five is Bacteroides
fragilis (see above); what are the other four?
_____________________________________
17. What is the DOC of Bacillus fragilis?
Your List of sources
for Listeria should
include cattle
products!
____________________________________
18. What is the virulence mechanism of Staph
aureus abscess formation?
Listeria is Cattle-ase
positive!
11. Is enterotoxin of Staph aureus heat stable or
labile? ______________________________
Food Poisoning
Incubation
Scombroid
Up to 1 hour
Staph aureus
Bacillus cereus
3 – 8 hours
Botulinum
½-day to 1-day
Salmonella enteritidis
1 – 2 days
1. Protein A: ___________________________
2. Coagulase: ___________________________
3. Leukocidin: ___________________________
This is not PCL! Actually it is ACL (“A” for Protein A!)
If you like PCL better, then P is for “Protein”!
Note: Protein A binds to Fc-IG and coats Staph with PMNs!
Rash on Palms and Soles
T
S
oxic Shock Syndrome (Staph aureus)
alded Skin Syndrome (Staph aureus)
Not that many diseases can cause rash on palms and soles. Solve
the puzzle by adding more bugs or diseases to the right column
until you figure out the hidden word of the first column. Note that the
answer will become apparent as we learn about more bugs.
11. Northwestern Medical Review, Bare Minimum Microbiology, 2012
19. What ionic situation is postulated to provide a
suitable medium for production of TSS toxin?
____________________________________
____________________________________
20 What gram-positive rod causes flaccid
paralysis and produces a heat labile toxin?
_____________________________________
21. Is salmonella toxin heat stable or labile?
____________________________________
About Scombroid Food Poisoning…
Scombroid food poisoning is a seaborne illness that often results
from eating decayed fish. It is one of the common types of seafood
poisoning; however because of its symptoms it is often classified as
or confused with type I hypersensitivity reaction (food allergy).
Symptoms, often self-limited, start within 10-30 minutes of ingesting
the fish and include skin flushing, erythema, abdominal
cramps, nausea, diarrhea, tachycardia, wheezing and hypotension.
It is proposed that histidine that is abundant in many types of fish is
converted to histamine via histidine decarboxylase in meat that is
stored at very low temperatures. Histamine is heat stable within
normal cooking temperatures, so even properly cooked fish can be
affected. Note that freezing, cooking and smoking do not destroy
the scombroid toxin. A few fish commonly poisoned with scombroid
include anchovy, bluefish, herring, mackerel, mahi-mahi, sardine,
and yellow fin tuna.
Diseases Caused By Staph aureus
S
OF
Skin Infections Pneumonia
For
TP
Sta
p
A
Osteomyelitis Acute Endocarditis h aureus INS
Dis
eas
es
Food Poisoning Infective arthritis
Toxic Shock Necrotizing Fasciitis
Syndrome
Sepsis
22. Clinical Settings for Toxic Shock
Syndrome:
A. _________________________________
B. _________________________________
C. _________________________________
23. What is the number 1 cause of necrotizing
fasciitis (flesh eating disease)?
_____________________________________
_____________________________________
24. What is the number 2 cause of death in noncoronary ICU patients and the 10th overall
cause of death in the USA?
_____________________________________
25. What are the major criteria used for diagnosis
of sepsis?
_____________________________________
_____________________________________
_____________________________________
_____________________________________
Hint: SIRS!
26. X-ray of the hip of an 8-year-old boy with
history of recurrent bone pain is significant for
avascular necrosis of the head of the femur. In
the past on three occasions the child has been
diagnosed with osteomyelitis. What organism
most likely might have caused this finding?
____________________________________
27. Would you change your answer if you knew
that the boy has sickle cell anemia?
___________________________________
28. Who am I?
 I am a must-know cousin of Staph aureus,
but I prefer to live on the skin!
 Often I like to dive into the bodies of 3
groups of people and roam through their
bodies. Docs say I give them “bacteremia.”
 You may ask “who are these three?” They
are the immunocompromised, hospital
patients with Foley urine catheters or IV
lines, and patients with prosthetic devices
such as valves or joints!
 I just don’t know what it means, but lab
technicians often say that I am a catalasepositive and coagulase-negative creature!
 I just hate vancomycin!
 Now can you tell me who am I?
________________________________
29. There are only three must know staph family
members for the exam. The most commonly
tested one is Staph aureus. The second one is
Staph epidermidis, which you were just
introduced to. Who is the third one?
__________________________________
Hint: It is the second most common cause of UTI (ranks after
E. coli) in ambulatory women.
Hint: It is coagulase negative!
12. Northwestern Medical Review, Bare Minimum Microbiology, 2012
Catalase NEgative
Catalase Positive
Answers
1. Penicillins bind to penicillin binding proteins (cell wall transpeptidases) and inhibit
cross-linkage of the peptidoglycan units of the bacterial cell wall.
2. Penicillinase
Staph
StrEp
Fibrinogen
Coagulase
4. Gram-positive!
Fibrin
Positive
Negative
3. Vancomycin inhibits cell wall synthesis in Gram-positive bacteria. In contrast to
penicillins that inhibit cross linkage of the peptidoglycan subunits, it acts by binding
to precursors of peptidoglycan subunits rather than by interacting with the
transpeptidases. It is proposed that it inhibits incorporation of N-acetylmuramic acid
and N-acetylglucosamine peptide subunits into the peptidoglycan matrix.
S. Aureus
S. Epidermidis
S. Saprophyticus
Which one is catalase
nEgative—Staph or Strep?
StrEp!
StAph is CAtAlAse positive!
5. The next DOC for MRSA is trimethoprim-sulfamethoxazole (Bactrim). Lately, there
has also been emergence of Vancomycin-Resistant Staph Aureus (VRSA). A few
newly developed antibiotics that can act on them are Linezolid, and daptomycin.
6. Staph is grape-like and Strep is chain-like!
7. PMNs!
8. Leukocidin is leukocidal and destroys PMNs!
9. Strep is catalase-negative and listeria is catalase positive
10. Listeria is catalase-positive!
11. Staph toxin is heat stable!
12. See the note about scombroid poisoning
13. 3 to 8 hours!
14. Salmonella food poisoning incubation is 1 to 2 days!
15. Bacteroides fragilis!
16. The other strict anaerobes are the four clostridial members (tetani, botulinum,
perfringens and difficile)
17. Clindamycin and metronidazole
CATA Co.
18. Protein A attracts PMNs; coagulase coagulates blood; and leukocidin kills the
neutrophils!
all staff members
In our company
love
cats!
Staff is
Catalase
Positive!
30. What are the top 3 bacterial causes of sepsis
as a result of skin infections?
____________________________________
____________________________________
____________________________________
31. Among -lactamase resistant penicillins that
can act on Staph aureus (NO MDs!) Nafcillin
is by far most preferable over Methicillin; why?
____________________________________
____________________________________
32. The treatment strategy of staph aureus and
Staph epidermidis endocarditis is similar
because of what important issue?
____________________________________
____________________________________
Note: Answer for “rash on palms and soles” will progressively become more apparent
as further bacteria are covered and we add in more words into the puzzle during the
course!
19. Hypomagnesemia!
It is postulated that tampons absorb various ions including magnesium.
Note: Staph aureus is a normal vaginal flora in 8-10% of females. But heavy
growth is unusual.
20. Clostridium botulinum causes flaccid paralysis
21. Salmonella toxin is heat labile
22. The 3 settings for TSS are tampons, nose-packing and wound dressing!
23. Strep pyogenes is the number one cause of necrotizing fasciitis. Other major
causes are Staph aureus, Clostridium perfringens and Bacteroides fragilis!
24. Sepsis is the number 2 cause of death in non-coronary ICU and 10th overall cause
of death in the USA.
Note: Recent data suggest that sepsis is the number one cause of death in noncoronary ICU
25. Criteria for diagnosis of sepsis: Presence of infection plus 2 or more signs of
Systemic Inflammatory Response Syndrome (SIRS): Tachycardia; Hypo or
hyperthermia; Tachypnea (hypocapnia); and WBC less than 4000 or more than
12000/mm3!
26. The most likely infectious cause of osteomyelitis is Staph aureus (also see answer
to question 27).
27. Children with sickle cell disease due to splenetic malfunction or splenectomy are
uniquely susceptible to frequent osteomyelitis as a result of salmonella infections.
Note: The cause of avascular necrosis is often vascular and aseptic; but it is
postulated that frequent infections and resultant inflammation of the bone may
cause stenotic damages to the vasculature, leading to avascular necrosis of the hip
and femoral head!
28. Staphylococcus epidermidis!
29. Staphylococcus saprophyticus!
30. Top 3 causes of sepsis as a result of skin infections are Strep pyogenes, Staph
aureus, and Pseudomonas!
31. Meticillin causes serious nephrotoxicity
32. Most Staph epidermidis strains are currently MRSA-like and they are resistant to
nafcillin!
13. Northwestern Medical Review, Bare Minimum Microbiology, 2012
Excerpts
ENDOCRINOLOGY
Adrenal Pathophysiology and Pharmacology
Part 1 of 3
14. Excerpts of Triology; Northwestern Medical Review, 2012
HYPOTHALAMIC-PITUITARY-ADRENAL AXIS
Positive CRH
Releasing Factors
Stress (via reticular formation); severe
hypoglycemia; anxiety/fear (via Amygdaloid
nucleus); increased ACh and serotonin; pain
(via ascending pathways); circadian rhythms
(via suprachiasmatic nucleus).
Negative CRH
Releasing Factors
• Increased GABA
• Increased enkephalins
• Increased opioids
Hypothalamus (CRH Cells)
CRH
Anterior Pituitary
ACTH
Adrenal Cortex
Cortisol
CRH via cAMP stimulates
synthesis of POMC (Proopiomelanocortin) which
splits into ACTH, MSH
and endorphin.
STEROID BIOSYNTHETIC PATHWAYS IN ADRENAL GLAND
Cholesterol
Desmolase
Pregnenolone
Hydroxysteroid Dehydrogenase
Progesterone
Steroid
Biosynthesis
17-hydroxylase
17-OH Progesterone
----------21-hydroxylase----------Deoxycorticosterone
Deoxycortisol
--------11-hydroxylase-----------
Corticosterone
Aldosterone
Mineralosteroids
Cortisol
Glucosteroids
Androstenedione
Testosterone
Estradiol
Sex Steroids
15. Excerpts of Triology; Northwestern Medical Review, 2012





ACTH Effects on the Adrenal Gland
ACTH acts on the adrenal cortex and causes
hypertrophy of zona fasciculata and zona
reticularis—sites of cortisol and androgen
precursor’s synthesis.
ACTH up-regulates ACTH receptors
Increases cholesterol uptake via HDL/LDL
receptors (increases adrenal free cytoplasmic
cholesterol levels).
Increases desmolase activity. Hence, increases
cholesterol conversion to pregnenolone.
Increases cytochrome P450 enzymes and causes
an increased conversion of pregnenolone to
cortisol and aldosterone.
Steroids
 Zona glomerulosa lacks 17 -hydroxylase, and it
is the only zone that contains aldosterone
synthase
 Zona glomerulosa has no enzymatic pathway to
synthesize
cortisol,
DHEA
(dehydroepiandrosterone) and androstenedione
 Male and female gonads lack 11 and 21 hydroxylase. Thus, gonads cannot produce
corticosterone, cortisol or aldosterone.
 DHEA and androstenedione can be converted to
testosterone in gonads.
 Adrenal’s testosterone synthesis is insignificant
 Gonads only possess enzymes that are required
for androgens (e.g. testosterone) and estrogen
synthesis.
ZONA GLOMERULOSA
CORTEX
(Mineralocorticoids-Aldosterone)
ZONA FASCICULATA
(Glucocorticoids--Cortisol)
ZONA RETICULARIS
(Gonadocorticoids--Sex steroids)
MEDULLA
(epinephrine, norepi.)
• Salt
• Sugar
• Sex
• Stress
The deeper you go the more excited you get!
Zona Glomerulosa makes the hormones that act on
the glomeruli—e.g. aldosterone!
In humans, sex starts with retinal stimulation!
Hence, zona reticulosa for sex hormones!
Cortisol is the stress hormone. Its blood level rises
just before waking up, and just before getting
ready to confront everyday’s challenges!





Cortisol
Binds to plasma proteins—Transcortin and
albumin
Plasma levels are high immediately prior to waking
and lowest around mid-night
Produced in Zona Fasciculata and Reticularis
Release is stimulated by ACTH and epinephrine
Signal transduction is via nuclear receptors (gene
transcription).
Metabolic Effects of Cortisol
 Prepares body for stress by metabolizing fuels
 Catabolism of muscle protein to provide substrate
for liver gluconeogenesis.
 Increases plasma glucose
Cortisol Effects on the Liver
 Increases amino acid (AA) uptake and
gluconeogenesis. Increases glucose export
(glucagon like!)
 Increases glycogen synthesis (insulin like!)
 Over all it has anabolic effect
Cortisol Effects on the Muscle
 Decreases glucose uptake, increases proteolysis
and increases AA export (amino acids go to liver),
decreases protein synthesis.
 Glucagon-like effect
 Catabolic role
Fasting drops blood sugar. Hence, zona fasciculata
for blood-sugar-raising hormone, that is, cortisol!
16. Excerpts of Triology; Northwestern Medical Review, 2012
Cortisol and the Adipose Tissues
 Decreases glucose uptake, increases lipolysis,
increases free fatty acid (FFA) export (FFAs go to
the liver and they are converted to acetylcholine,
the precursor of ketone bodies) and increases
plasma glycerol level.
 Catabolic role
Anti-Inflammatory Roles of Cortisol
 Strong inhibition of the entire inflammatory
response
 Prevents transplant rejection
 Decreases
neutrophil
recruitment,
and
prostaglandin and leukotriene synthesis by
inhibiting phospholipase A2
 Decreases T-cell count, and inhibits production of
interleukin-2.
 Decreases capillary permeability, and inhibits
histamine release from mast cells.
 Shrinks the lymphoid mass and drops the
lymphocytic count. Hence, it is desirable in
treatment of lymphomas.
Note: Patients with Cushing’s syndrome present with
shrunken lymphoid mass. Prednisone (a glucocorticosteroid) is one of the commonly used drugs in
treatment of Hodgkin’s lymphoma.
Potency of Various Steroids
 All steroids have both mineralo and glucocorticoid
effects.
 Cortisol and cortisone have equal mineralo and
glucocorticoid effects.
 Dexamethasone by far is the most potent
glucocorticosteroid, and fludrocortisone is the
most selective mineralocorticoid.
Fludrocortisone is Fluidocortisone!








Glucocorticoid (Cortisol) Side Effects
Hypokalemic alkalosis (mineralocorticoid effects)
Glycosuria (Cushing’s diabetes)
Increased susceptibility to infection
Peptic ulcers
Myopathy
Osteoporosis
Cushing’s symptoms and hypertension
ACTH suppression
2. The management of myasthenia gravis requires
corticosteroid therapy. However, at times the
patients are presented with worsening
symptoms; why?
_______________________________________
_______________________________________
Phospholipase A2
Corticosteroids
Arachidonic Acid
Cyclooxygenase
Lipoxygenase
NSAIDS
Decreasing Order
Membrane Phospholipid
Potency of Glucocorticoids
Dexamethasone
Cortisol
Corticosterone
DECORTIzation order!
Leukotrienes
Increase Bronchial Tone
Prostacyclin
Prostaglandins
Thromboxane
• PGI2
• Decrease vascular &
bronchial tone
• Increase Platelet
aggregation
• Decreases Platelet
aggregation
• Increase Uterine Tone &
bronchial tone
• Increase vascular &
bronchial tone
• Decreases vascular,
uterine & bronchial tone
Please refer to chapter on eicosanoids and NSAIDs for more
information on the above diagram.
1. What non-steroid hormone has intra-nuclear
transduction?
______________________________________
Anti-Steroids
 Aminogluthemide
 Inhibits P-450 and in turn inhibits desmolase
activity
 It is used in (1) Cushing’s treatment and (2) to
reduce estrogen level in breast cancer
 Ketoconazole:
Desmolase
inhibitor and
antifungal. Reduces all steroids.
 Spironolactone: Diuretic used for treatment of
hypertension and primary hyperaldosteronism.
Note: Aromatase converts androgens to estrogens and promotes
estrogen formation.
17. Excerpts of Triology; Northwestern Medical Review, 2012
ADRENOCORTICAL DISEASES (PART I)
Duration of Action of Glucocorticoids
Short Acting Glucocorticoids
 Hydrocortisone and cortisone
 Duration of action: 8-12 hours
Intermediate Acting Glucocorticoids
 Prednisone, prednisolone, triamcinolone
 18 - 36 hours
Long Acting Glucocorticoids
 Betamethasone, dexamethasone, paramethasone
 1 - 3 days hours



Potency of Various Steroids
All steroids have both mineralocorticoid and
glucocorticoid effects
Cortisol has equal mineralo and glucocorticoid
effects
Dexamethasone is by far the most potent
glucocorticoid, and fludrocortisone is the most
selective mineralocorticoid.
Case for the Next 3 Questions
A 17-year-old girl is being admitted to the OB/GY
clinic for the complaint of amenorrhea. Examination
reveals lack of development of secondary sex
characteristics. Blood pressure is 180/110; result of
funduscopy is normal; she has no lymphadenopathy
or hepatosplenomegaly. Blood analysis indicates
hypokalemia, hypernatremia, metabolic alkalosis, and
suppressed renin. Urinary analysis confirms a high
level of urinary gonadotropins.
6. Suppression of rennin in this patient is due to
which of the following options?
(A)
(B)
(C)
(D)
(E)
Negative aldosterone feedback
Insensitivity of kidney to aldosterone
Lack of adrenal development
Lack of ACTH
Insensitivity of JG cells
7. Which of the following hormones is expected to
increase in this patient’s plasma?
(A) Deoxycortisol and cortisol
(B) Deoxycorticosterone and corticosterone
(C) Progesterone and deoxycortisol
(D) Androstenedione and corticosterone
(E) Aldosterone and cortisol
3. Three steroids with noticeable mineral effects:
A_____________________________________
F_____________________________________
D_____________________________________
8. Which of the following deficiencies is the most
likely cause of this patient’s problems?
(A)
(B)
(C)
(D)
(E)
4. What are the top two side effects of mineralocorticoids?
_____________________________________
_____________________________________
5. Cushing’s and Addison’s disease have opposing
effects on the level of eosinophils. Which one
causes eosinophilia and which one causes
eosinopenia?
_______________________________________
11-hydroxylase
21-hydroxylase
17-hydroxylase
Desmolase
Hydroxysteroid dehydrogenase
Hydroxysteroid Dehydrogenase Deficiency
Congenital adrenal hyperplasia
Lack of steroids removes negative feedback over
the pituitary and ACTH production stays
uninhibited. Constant ACTH production causes
hypertrophy of the adrenal cortex.
 Pregnenolone is not converted to all other
subsequent steroids.
 All pathways; mineralocorticoids, corticosteroids
and glucocorticosteroids


18. Excerpts of Triology; Northwestern Medical Review, 2012


The second most fatal steroid biosynthetic
deficiency.
Early death
Note: Desmolase deficiency is by far the most fatal
steroid deficiency. Many patients die in early infancy.
Cholesterol
Desmolase
Pregnenolone
Hydroxysteroid Dehydrogenase









Deficiency of hydroxysteroid
dehydrogenase is associated with
----------21-hydroxylase----------increase
in pregnenolone and total
deficiency of all steroids and products
after the affected step.
21-Hydroxylase Deficiency
The most common deficiency.
Congenital adrenal hyperplasia
Deficiency of both mineralocorticoids and
glucocorticoids.
Due to lack of feedback over pituitary there is a
high level of ACTH in the plasma.
Pathways are shunted to sex steroid production
There is a high level of androgens
Adrenal virilism. Male phenotype exaggerated.
Female patients present with masculine
characteristics such as enlarged clitoris
Salt loss and hypotension
Cholesterol
Desmolase
Pregnenolone
Hydroxysteroid Dehydrogenase
 Progesterone
Progesterone
17-hydroxylase







17-Hydroxylase Deficiency
Autosomal recessive defect
Congenital adrenal hyperplasia
Conversion of progesterone to subsequent sex
hormones and glucocorticoids is affected.
Female characteristics are preserved, but
secondary developments are affected.
Pathways
are
shunted
to
further
mineralocorticoids production.
Increased mineralocorticoids leads to sodium
and water retention and hypertension
Due to lack of androgens (i.e. development of
zona reticularis) female phenotype is
exaggerated. For instance males have shrunken
external genitalia.
Cholesterol
Desmolase
17-OH Progesterone
Androstenedione
21-hydroxylase
Deficiency
Estradiol
Zonal Glomerulosa



Pregnenolone
Hydroxysteroid Dehydrogenase

Progesterone
17-hydroxylase
---------------- 21-hydroxylase ----------------
Testosterone

Zona Fasciculata
www.northwesternmedicalreview.com
11-Hydroxylase Deficiency
Autosomal recessive deficiency that causes
deficiency of corticosterone, aldosterone and
cortisol.
Increased level deoxycorticosterone leads to salt
and fluid retention, and hypertension.
Hypertension is due to accumulation of
deoxycorticosterone that has a very strong
mineralocorticoids effect.
Like 21--hydroxylase leads to adrenal virilism
(AKA.
adrenogenital
syndrome)
and
masculinization. More common in males.
Less common than 21-hydroxylase deficiency
Deoxycorticosterone
---------------- 11-hydroxylase ---------------Corticosterone
17-hydroxylase Deficiency
Aldosterone
Zonal Glomerulosa
Zona Fasciculata
www.northwesternmedicalreview.com
Zona Reticularis
Zona Reticularis
19. Excerpts of Triology; Northwestern Medical Review, 2012
Cholesterol
Desmolase
Pregnenolone
But the most commonly remembered one on the
history of the exam (i.e. the most commonly tested
one) has been the 17-hydroxylase deficiency!
Progesterone
17-hydroxylase
17-OH Progesterone
Testosterone
11-hydroxylase deficiency
Zonal Glomerulosa
Drill and Self-Study Questions
Androstenedione
Deoxycorticosterone
Zona Fasciculata
www.northwesternmedicalreview.com
Estradiol
Zona Reticularis
Blackjack Makes You Happy (Hypotensive!)
9. In the following two pedigree diagrams the solid
squares and circles indicate genetically affected
individuals. Which one of them is most likely
affected with 17-hydroxylase deficiency?
A.
B.
C.
D.
E.
A
B
C
A and B
B, C and D
About the Black Jack Mnemonic!
The top three tested congenital steroid deficiencies are
21, 17 and 11-hydroxylase deficiencies.
All three
conditions are associated with hypertrophy of the
adrenals due to high levels of ACTH (lack of
suppression).
The highest number among the three is 21. This number
happens to be the highest and mostly desired number in
black jack as well. It is the only number that if you get it
in the game you would be happy (hypotensive). The other
two numbers in the game, quite often will make the
players hypertensive; especially if the dealer shows a 10
or 9 up!
In short, 11 and 17 hydroxylase deficiencies are
associated with hypertension whereas 21-hydroxylase is
associated with hypotension.
Important Note: Hypertension is the top most
important distinguishing characteristic between 11
and 21-hydroxylase deficiencies. At the same time,
the most common deficiency among all congenital
adrenal deficiencies is the 21-hydroxylase deficiency.
10. A newborn presents with ambiguous external
genitalia. The genitalia seem more like an
enlarged clitoris than penis. There is a scrotum–
like structure that has resulted from labial fusion.
Ultrasound confirms normal ovarian development
and presence of bilaterally enlarged adrenal
glands. Karyotyping indicates a 46 XX karyotype.
Of the following conditions which one is the most
likely cause of these findings?
A. 11 alpha-hydroxylase deficiency
B. 17 alpha-hydroxylase deficiency
C. 21 alpha-hydroxylase deficiency
D. Desmolase deficiency
E. Turner’s syndrome
20. Excerpts of Triology; Northwestern Medical Review, 2012
11. At age 10 the patient is admitted to the hospital
for complete physical evaluation. Which of the
following lab findings would be the most
remarkable finding in the patient?
A. High level of cortisol
B. Low level of ketosteroids
C. Low blood pressure
D. Low level of rennin
E. Low level of ACTH
12. Addison’s disease causes eosinophilia and
darkened mucosa. What are the major causes of
eosinophilia?
_____________________________________
_____________________________________
13. Wegener’s Granulomatosis also causes
eosinophilia. What are some of the main
characteristics of this disease?
_____________________________________
_____________________________________
14. What is the name of the condition that is
associated with eosinophilia, fibrotic thickening of
heart valves and restrictive cardiomyopathy?
_____________________________________
_____________________________________
Answers
1.
The only other hormone other than steroids with
intracellular transduction is thyroxine.
2.
Steroid-induced myopathy is the problem associated
with myasthenia treatment. Despite this finding
corticosteroids are the mainline medications.
Note: MG is an autoimmune disease and often is
associated with thymoma.
3.
The 3 steroids that have mineralocorticoid effect are:
aldosterone, fludrocortisone and deoxycorticosterone.
4.
Side effects of mineralocorticoids are hypokalemia
and congestive heart failure (CHF) due to volume
overload!
5.
Cushing’s (hypocortisolism) depresses immune
functions and causes eosinopenia. Addison (lack of
steroids) does the opposite (eosinophilia).
6.
Option [A] is correct.
7.
Option [B] is correct.
8.
Option [C] is correct.
9.
The correct answer is [A]. 11, 21 and 17 alphahydoxylase deficiencies are all autosomal recessive
conditions. Only the pedigree I is illustrative of
autosomal recessive conditions. Hence, the female
person marked “A” is the one who is affected with the
deficiency. Please note that it is very unlikely for
patients with 17-hydoxylase deficiency to get
pregnant. But with rigorous hormonal therapy (e.g.
estrogen and glucocorticosteroid therapy, and in vitro
fertilization) this may not be impossible.
Also note that the pedigree II is illustrative of dominant
genetic conditions. The person “B” in this pedigree has
randomly acquired a mutated dominant gene.
10. The correct answer is [C]. Patients with 21
hydroxylase deficiency present with adrenal virilism.
As a result females present with ambiguous and
enlarged clitoris, and males present with exaggerated
genitalia and precocious puberty. These patients
characteristically are hypotensive as a result of lack of
development of zona glomerulosa or production of
mineralocorticoids.
Note that as result of low
aldosterone and blood volume and pressure, the
rennin level is going to be high in these patients. Also
as a result of lack of cortisol, the level of ACTH is also
high in the patient. Also note that ketosteroids are the
byproducts of metabolism of androgens such as
testosterone or dihydrotestosterone. The patients with
21-hydroxylase deficiency have a very high level of
androgens that raises the levels of ketosteroids in
their serum and urine.
11. Option [C] is the correct answer. The karyotype of
46XX indicates that the patient has acquired
congenital adrenal virilism (masculinization) as a result
of too much androgens or over-activity of zona
reticularis. Note, that the option “A” in question 10 may
also cause the very same symptoms. But by far 21hydroxylase deficiency is the most common cause of
congenital adrenal hyperplasia, and between the two
options in question 10 it seems to be the more
probable answer. Note that 11 hydroxylase deficiency
in contrast to 21 hydroxylase causes hypertension.
12. “NAACP“is the mnemonic for major causes of
eosinophilia (see the next mnemonic)!
21. Excerpts of Triology; Northwestern Medical Review, 2012
Causes of Eosinophilia
Note: Eosinophilia is often
associated with skin
hypersensitivity reactions
that color the skin. In
Addison’s, ACTH is high and
as such MSH is high
13. About Wegener’s Granulomatosis…
(Please review the chapter on vascular pathology for more information)
 Granulomatous inflammation of arterioles, capillaries
and venules.
 Triad of Wegener’s: (1) Focal lung vasculitis; (2)
Necrotizing pulmonary granuloma; and (3) Necrotizing
glomerulitis.
 Fifth decade onset. 80% of patients die in year one if
not treated
 Immunosuppression (with cyclophosphamide) has
saved more than 90% of the victims.
 C-ANCA (antineutrophilic cytoplasmic autoantibodies)
is present in more than 90% of patients. It is a good
disease marker.
14. Loeffler’s endocarditis (A.K.A. endomyocardial
fibrosis)
(Please review Bare Minimum Pathology Book for more information)
**End of Adrenal Endocrinology Part I**
22. Excerpts of Triology; Northwestern Medical Review, 2012
Excerpts
Triology Book
Physiology and Pharmacology with Relevant Pathology
Aortic Stenosis:
Ventricular Pressure-Volume Loop Pathophysiology
23. Excerpts of Triology; Northwestern Medical Review, 2012
Pressure-Volume Loop: Aortic Stenosis (AS)
A 77-year-old woman is presented with syncope
during her routine daily exercise. Her history is
significant with several other episodes of syncope
within the past 6 months. She has a systolic murmur
that is best heard at the right upper sternal border.
There is radiation of the murmur into the neck. Which
of the following options is the most probable
diagnosis?
(A) Mitral stenosis
(B) Mitral regurgitation
(C) Tricuspid regurgitation
(D) Aortic stenosis
(E) Aortic insufficiency
1. What is the second most common valvular
anomaly and the second most commonly tested
valvular defect? ________________________
Note: Over time the
loop may tilt a little to
the
right.
But
concentric hypertrophy
will oppose its overexpansion!
2. What happens to stroke volume in AS and what
portion of the shaded diagram in the left
represents it?
_______________________________________
_______________________________________
3. What will happen to end-systolic ventricular
volume (afterload) in AS?
_______________________________________
_______________________________________
4. Do you expect the velocity of fiber–shortening
increase or decrease in AS, or any condition that
increases the afterload?
_______________________________________
_______________________________________
5. What is the expected end diastolic volume in AS,
and how does this affect the force of contraction?
_______________________________________
_______________________________________
The above shaded P-V loop represents mild to severe aortic
stenosis. Compared to normal heart, the ventricular pressure
rises sharply in systole; meanwhile less volume is ejected into the
aorta.


Impairment of Cardiac Output
Aortic stenosis (AS) impairs ventricular emptying
due to high outflow resistance. A large pressure
gradient (see the next picture) is required to push
blood through the valve; this raises the peak (or
end-systolic pressure or the afterload).
Notice the wide difference between intraventricular pressures of 200/30 mmHg versus
intra-aortic pressures of 90/70 in AS.
6. What is the key adaptive (structural cardiac
remodeling) response to AS?
_______________________________________
_______________________________________
7. What is the expected systemic blood pressure in
AS?
_______________________________________
_______________________________________
8. What is an important clinical finding in AS?
_______________________________________
9. What type of dysfunction would you expect to
see in AS; systolic or diastolic?
_______________________________________
24. Excerpts of Triology; Northwestern Medical Review, 2012
10. Patients with AS are presented with angina.
What is the pathophysiology of angina in AS?
_______________________________________
11. What is a reasonable explanation for syncope of
the AS patients during exercise?
_______________________________________
_______________________________________
12. The following diagram shows the characteristic
auscultation findings of the heart of a patient
diagnosed with AS. What is this finding?
_______________________________________
13. What are the two unique auscultation findings in
paradoxical split?
_______________________________________
_______________________________________
“A” is always before “P”
Except in PA radoxical split; wherein
P2 is before A 2 (as in the word “Paradox”)!
14. What are the top 3 causes of paradoxical split?
_______________________________________
_______________________________________
_______________________________________
15. In which of the above conditions the afterload
decreases?
_______________________________________
_______________________________________
This astronaut was the captain in charge of
the last NMR-space mission that was
aborted prematurely. Since then NMR has
categorically halted all extraterrestrial
missions. Why?
Hint: Watch the last 30 seconds of the videotape that
accompanies this lecture.
_______________________________________
_______________________________________
Bicuspid valves calcify at an earlier age compared to
trileaflet valves. Age of onset of symptoms in calcified
valves is 45 for bicuspid and 75 years for trileaflet
valves.
17. What is the best way to evaluate the function of
the stenotic valve?
_______________________________________
_______________________________________
18. What aortic valve size (area) is considered
severe stenosis?
_______________________________________
19. Normally aortic and pulmonary valves have three
leaflets (cusps). Why do they call them semilunar
valves?
_______________________________________
_______________________________________
20. Do you expect patients with aortic stenosis to
have wide or narrow pulse pressures?
_______________________________________
21. What is the best location to auscultate the aortic
valvular sounds?
_______________________________________
22. List the must-know clinical findings and keywords
of aortic stenosis:
_______________________________________
_______________________________________
_______________________________________
_______________________________________
_______________________________________
_______________________________________
Why do patients with
aortic stenosis look
so SAD?
16. List the common causes of AS from the most to
the least common?
_______________________________________
25. Excerpts of Triology; Northwestern Medical Review, 2012
Answer:
They look sad because “sad” stands for:
S yncope
A ngina
D yspnea!
23. Aortic stenosis is more common in elderly
women as a result of age-related calcification of
the valve. However, prevalence of the disease in
young-age is more common in males and it is
associated with bicuspid (instead of tricuspid)
aortic valves. What would be a reasonable
explanation for this finding?
_______________________________________
_______________________________________
Note: AS reminds you of Astronaut and the rocket
reminds you of the shape of the P-V loop in AS.
Also, the tip of the rocket reminds you of the
ventricular pressure that propels the rocket!
26. Excerpts of Triology; Northwestern Medical Review, 2012
ANSWERS
1.
The second most common Valvular defect is aortic
stenosis. This also happens to be the answer to the
cited case scenario.
2.
The stroke volume is represented by the width of
the loop and it is decreased in AS.
3.
End-systolic ventricular volume increases in AS!
4.
Increased afterload slows the velocity of myocytic
shortening. Because the period of time available for
ejection is fixed at about 200 msec, a decrease in
fiber shortening velocity drops the volume ejection
and as a result more blood remains in the ventricle
after systole.
5.
As a result of increase in end-systolic volume the
excess residual blood is added to the incoming
venous blood and raises the EDV. This raises the
preload and as a result (via Starling Law) the force
of contraction. Note that left ventricular concentric
hypertrophy will limit the over-expansion of the enddiastolic volume (to the right).
to exercise and as a result blood perfusion to
their brains is drastically affected.
12. Paradoxical split of the second heart sound.
13. Systolic crescendo-decrescendo murmur and
paradoxical split.
14. Top three causes of paradoxical split: Aortic
stenosis, left bundle branch block, and systemic
hypertension.
15. In aortic stenosis and systemic hypertension
16. Age-related calcification of the valve, congenital
bicuspid valve, and acute rheumatic fever
17. The best way to evaluate the function of a stenotic
valve: Electrocardiography.
6.
Concentric ventricular hypertrophy!
18. Less than 1 cm2
7.
As a result of a fall in stroke volume, the arterial
pressure drops substantially in AS.
19. Cross-section of the valves (the three leaflets) has
semilunar appearance.
8.
Syncope and faintness is a common clinical finding
in AS.
9.
Diastolic dysfunction due to hypertrophied left
ventricle
20. Pulse pressure is equal to systolic minus diastolic
pressure. As a result of low systolic pressure, AS
patients have a narrow pulse pressure
10. Angina of AS is due to left ventricular hypertrophy
and as result increased oxygen demand of the
heart.
11. AS causes a low cardiac output. During exercise
vasodilation of the skeletal muscles decreases
peripheral vascular resistance. In normal individuals
this decreased resistance is compensated for by an
increase in the cardiac output. Patients with severe
AS cannot increase their cardiac output in response
21. Second intercostal space to right of the sternum.
aoRTic = RighT !
22. Must-know clinical findings of AS: Paradoxical split,
Systolic crescendo-decrescendo murmur, narrow
pulse pressure, dyspnea upon exertion, and
syncope (faintness)
23. Congenital bicuspid aortic valve is inherited in an xlinked fashion. Also it has more tendencies for
calcification!
27. Excerpts of Triology; Northwestern Medical Review, 2012
Excerpts
Bare Minimum
Neurology and Neuroanatomy Review
Visual Field Defects
28. Northwestern Medical Review, Bare Minimum Review Series, 2012
LESIONS OF THE VISUAL PATHWAY
Normally damage to the brain cortex causes
contralateral deficits in the extremities. This is not,
however, true of the visual system. Unilateral damage
to the visual cortex is manifested by characteristic
partial loss of vision in both eyes. Neither of the eyes is
able to see the contralateral visual field with respect to
the location of lesions. As we will see later this unique
pattern also holds true for the oculomotor muscles. In
this case either of the eyes will have difficulty in moving
to the contralateral position.
Left
T
The next drawing illustrates the visual field (field of
view) through the left and right eyes. To better
understand the diagram and concepts that will follow
hereafter it would be helpful to suppose that you are
looking over the head of the perceiver and through
his/her eyes. The visual field would then be what the
perceiver actually would be able to see in front of him or
her.
N
T
Right
Visual Field
Temporal Retina
Nasal
Retina
Fovea
Optic Nerve
Chiasm
Optic Tract
Lateral Geniculate
Parietal Lobe Optic
Radiation (From Superior Retina)
Temporal Lobe Optic
Radiation (From Inferior Retina)
Calcarine Fissure
Following diagram depicts report of the patient to her
physician about how she sees the world (i.e. visual field)
through her right eye.
“DOC a pie is missing on the
lower inner side of whatever
I look at!”
Visual Cortex
 This tumor most frequently occurs in the convexities
of the cerebral hemisphere, parasagittal/falx-cerebri,
olfactory area, and supra-sellar region.
It arises from the arachnoid’s layer.
It is more common in woman (3:2 female-male
ratio); and often happens between ages 20 to 60.
 It is the second most common primary brain tumor.
It is benign and grows slowly
Histologically it is characterized by uniform whorling
pattern and calcified psammoma bodies.
 Surgical removal is quite successful.
 Associated
with
bitemporal
heteronymous
hemianopia.
 This tumor is ___________________________
Note: Suprasellar means above the sella turcica (literally Turkish
Saddle). It is the saddle-shaped depression in the sphenoid
bone that houses pituitary gland.
29. Northwestern Medical Review, Bare Minimum Review Series, 2012
About Calcarine Fissure
The calcarine fissure (or calcarine sulcus) is an
anatomical landmark located at the caudal and medial
surface of the occipital lobe and divides the visual
cortex into upper and lower parts. Visual information
that originates on the upper retina is projected to the
visual cortex above the calcarine, and those that
originate in the inferior retina, to the inferior part of the
calcarine fissure.
Common Primary Brain Tumors
1. The most common primary brain tumor is:
_______________________________________
8. In the above diagram, circular structures that are
labeled “A” and “B” represent tumors of the brain.
One of the two tumors belongs to a 5-year-old child
and the other to a 70-year-old man. Which one is
which?
_______________________________________
_______________________________________
9. Also called “Blue Cell Tumor”, this brain tumor
commonly occurs in children. The most common
location for it is the midline of cerebellum. It may
spread into the CSF. What is this tumor?
_______________________________________
2. The second most common primary brain tumor is:
_______________________________________
3. The third most common primary brain tumor is:
_______________________________________
4. Gliomas are tumors that originate in the brain or
spine. They are named according to the types of
cells that they originate from. What are the four
main cell types of gliomas?
_______________________________________
Pathways
that
originate
in
the
superior or inferior
retina are projected
to the superior or
inferior aspects of
the calcarine fissure
respectively!
Parietal Lobe Radiations
_______________________________________
(From Superior Retina)
5. What is the most common astrocytoma?
_______________________________________
6. Name the two most important nerve sheath
tumors?
_______________________________________
7. In the diagram below the exaggerated (caricature)
crescent structure (labeled “?”) is a fold of dura
mater forming a roof over the posterior cranial
fossa, and separating the cerebellum from the
basal surface of the occipital and temporal lobes of
the cerebral cortex. What is the term that correctly
describes this structure?
_______________________________________
Temporal Lobe Radiations
(From Inferior Retina)
Inferior to Calcarine Fissure
About Psammoma Bodies



AKA. Sand bodies
Mineralized (calcium deposit) bodies composed
usually of a central capillary surrounded by
concentric whorls
They can occur in benign and malignant epithelial
tumors (such as papillary ovarian or thyroid
carcinoma, and meningiomas)
Papillary carcinoma of the thyroid
Serous ovarian cystadenocarcinoma / Sand tumor
Adenocarcinoma of ovary
Meningioma
Mesothelioma
Omatous-like lesions
Malignant or benign
Adenocarcinoma
30. Northwestern Medical Review, Bare Minimum Review Series, 2012
Doc, what’s happening to
me; I can’t see my shoes;
I can’t see my belt; and I
can’t even see my “youknow-what!”. I can’t see
anything from my nose to
toes!
Doc: “We will get back to you a little later, just be patient
for now!”
Note: The simplified method of illustrating the visual
pathways, above, is very helpful in learning the lesions of the
pathways hereafter.
Beauty is in the eye of
beholder!
World through Bitemporal
Heteronymous Hemianopsic eyes
Human mind is a powerful tool.
Quite often it fills up the
perceptual gaps as it pleases !
12. This patient has a tumor that
has severed the temporal
retinal pathways on the rightside and has spared the
decussating fibers. Name and
draw the visual defect of the
patient.
________________________________________
________________________________________
10. Name the term used to describe the above visual
finding in lesions of chiasm?
_______________________________________
11. What are a few major causes of the above finding?
________________________________________
_______________________________________
Constructing Simplified Visual Pathways
13. In the following diagram there are two lesions
(severance of pathways); one before, and the
other after the lateral geniculate body. In either
condition the patient reports similar visual field
deficits. What is the descriptive term for this
condition, and how would you illustrate it by filling
out the diagram below?
_______________________________________
_______________________________________
31. Northwestern Medical Review, Bare Minimum Review Series, 2012
14. A patient with pituitary
adenoma has lesions of the
visual pathway at the level of
chiasm that has uniquely
spared the decussating fibers.
Name and draw the visual
defect of the patient.
_______________________________________
_______________________________________
18. A patient with recent history
of deep vein thrombosis
presents with thrombosis of
the basilar artery at the
bifurcation of the two
posterior
cerebral
circulations. As a result he
has bilateral damage to his
visual cortex (as it is shown
in the diagram). Name and
draw his visual field defect?
_______________________________________
_______________________________________
15. What lesions cause homonymous defects?
_______________________________________
_______________________________________
Note: Patients with end stage glaucoma in one or both
eyes may present with unilateral or bilateral macular
sparing.
16. What is the distinctive
difference
between
severing
projection
pathways at the position
(A) and severing them
after
the
lateral
geniculate (B)?
_______________________________________
_______________________________________
17. A patient has been
diagnosed
with
metastatic brain tumors.
The
tumors
have
severed his visual
pathways as it is shown
in the diagram. Name
and draw his visual field
defect?
_______________________________________
19. What is the following picture all about?
_______________________________________
_______________________________________
Note: As a general rule lesions that are located farther away
from the lateral geniculate bodies and closer to the visual
cortex are presented with a more prominent macular
sparring.
32. Northwestern Medical Review, Bare Minimum Review Series, 2012
Superior and Inferior Visual Fields
Above diagram shows location of the lesion in the temporal lobe of
the patient in the former question in a lateral view of the brain.
21. We have talked so many times about lateral
geniculate nucleus or body (LGN). What is this
organ; what is its function and where is it located?
_______________________________________






Superior visual field information is projected onto
the inferior retina, and the inferior field information
onto the superior retina
The neurons that originate in the superior or
inferior retina maintain their topographic positions
throughout the visual pathway until they reach the
visual cortex.
Superior retinal neurons pass through the parietal
lobes and end on the superior aspects of the
calcarine fissure.
Inferior retinal neurons pass through the temporal
lobes and end on the inferior aspect of the
calcarine fissure
From retina through the lateral geniculate bodies
the superiorly and inferiorly located neurons are so
close to each other that a lesion will sever both of
them together.
Past the lateral geniculate, the optic radiations
through the parietal or temporal lobes are spatially
apart from each other. As a result a lesion may
selectively sever one of the two radiations and
spare the other.
20. You are evaluating a
patient with lesion of the
left temporal lobe due to
vascular thrombosis. What
is the name of the
characteristic visual field
defect of the patient, and
how would you draw it?
_______________________________________
_______________________________________
_______________________________________
22. Draw and name the
visual defect that results
from severance of the
temporal radiations in
this picture
_______________________________________
_______________________________________
23. Draw and name the visual
defect that results from
bilateral severance of the
parietal lobe radiations in
this picture.
_______________________________________
_______________________________________
24. What type of cerebral
lesions will cause the
following visual field
finding?
_______________________________________
_______________________________________
33. Northwestern Medical Review, Bare Minimum Review Series, 2012
25. Using the following diagram and lesions draw and
name the visual defect of the following patient?
30. A patient is presented with the following visual field
defect pattern. Which of the following options better
explains this finding?
A. Right eye blindness and bitemporal
heteronymous hemianopia
B. Bitemporal heteronymous hemianopia and left
homonymous hemianopia
C. Neither of the above
D. Both of the above
_______________________________________
_______________________________________
26. What lesions of the visual pathways cause
homonymous defects?
_______________________________________
_______________________________________
Use the word “hetero” if the defect seen by the left and
right eyes are not to the same side of the visual field
(i.e. one is to the left and one to the right). Use the word
“homo” if the visual field defects of the left and right
eyes are to the same side (i.e. either on the left or right.
27. Which of these two findings is a
heteronymous and which one a
homonymous defect?
_________________________
_________________________
28. This picture is the left eye report
of a 25-year-old female patient
during
her
ophthalmologic
examination. Name the visual
defect and the most likely
disease that she is suffering
from?
_______________________________________
_______________________________________
29. What is the mechanism of macular sparing?
_______________________________________
_______________________________________
34. Northwestern Medical Review, Bare Minimum Review Series, 2012
Answers
1.
The top most common Glioblastoma multiforme
2.
The second most common: Meningioma
3.
The 3rd most common: schwannoma.
4.
The four main cells types and tumors are ependymomas
(ependymal
cells),
astrocytomas (astrocytes),
oligodendrogliomas (oligodendrocytes) and nerve
sheath tumors.
5.
The most common (and most aggressive) astrocytoma
is glioblastoma multiforme.
6.
The two major nerve sheath tumors are neutrofibromas
and schawannoma
19. “This is the picture of a pie in the sky!”
7.
The crescent structure is tentorium cerebelli
20. Right superior (upper) quadrantanopia. Pie in the sky!
8.
Adult tumors are often supra-tentorial (i.e. “A”) whereas
children tumors are often infra-tentorial (i.e. “B”).
9.
“Blue Cell” brain tumor of children is medulloblastoma.
10. Bitemporal heteronymous hemianopia!
11. Pituitary Adenomas, craniopharyngiomas, and
meningiomas.
12. Right nasal heteronymous hemianopia. Common
causes are calcified right internal carotid arteries and
compression of the chiasm.
13. Left homonymous hemianopia
18. The patient with bilateral damage to visual cortex due to
lack of circulation presents with has bilateral macular
sparing (AKA. bilateral homonymous hemianopia with
macular sparing)
21. The left and right lateral geniculate nuclei (LGN) are
within the thalamus and they are the primary relay
centers for visual information from the retina. Each LGN
receives information from one half of the visual field.
For example, the left LGN receives information from
ganglion cells of the temporal retina of the left eye plus
information from the nasal retina of contralateral (right)
eye. The nerves that originate in the retina have long
axons that extend, uninterrupted, through the optic
nerve, chiasm and optic tract and reach to the LGN. In
the LGN they synapse with the second set of neurons
that extend to the visual cortex.
22. Bilateral superior hemianopia or superior homonymous
hemianopia. Of course, there may also be a macular
sparing.
14. Binasal heteronymous hemianopsia
23. Inferior (Lower) (Homonymous) hemianopia
15. Lesions after the chiasm (and excluding chiasm cause
homonymous defects. Lesions before the chiasm cause
heteronymous defects
16. Both lesions cause right homonymous hemianopia.
However, the lesion in “B” causes right homonymous
hemianopia with macular sparing.
24. Vascular damage to the occipital visual cortex (example
bilateral occlusion of the posterior cerebral circulation)
25. Right inferior (homonymous) quadrantanopia and left
superior quadrantanopia.
17. He has bilateral macular sparing (AKA. bilateral
homonymous macular sparing).
35. Northwestern Medical Review, Bare Minimum Review Series, 2012
26. Lesions anywhere behind the chiasm; optic tract, lateral
geniculate, optic radiations, and visual cortex cause
homonymous defects!
27.
28. This condition, often unilateral, is called central scotoma
seen in optic neuritis and it may also result from optic
disc swelling. It is commonly associated with multiple
sclerosis.
29. Due to the large size of foveal representations in the
cortex, the areas that are devoted to central vision
(central parts of the visual field) are less likely to be
affected. Additionally, it is shown that the occipital lobe
receives a dual blood supply. Even though the posterior
cerebral artery is the main supply, a branch of middle
cerebral artery also supplements the occipital pole areas
that are topographically represented by the macula.
30. The best option is “D”. When you superimpose the
following two diagrams you will generate the picture that
is shown in the tested item.
36. Northwestern Medical Review, Bare Minimum Review Series, 2012
Carbohydrate Metabolism
Synopsis of Glycolytic Enzyme Deficiencies
The video that accompanies the texts in this segment begins on page 44. However, a
quick review of the rest of the materials is highly recommended prior to watching the
video.
37. Northwestern Medical Review, Bare Minimum Review Series, 2012
Glycolytic Pathway
Glycogen
Glucose 1-phophate
Glucose
Glucose 6-phosphate
Fructose 6-phosphate
Fructose 1,6 bisphosphate
Dihydroxyacetone phosphate
Glyceraldehyde 3-phosphate
1, 3-Diphosphoglycerate
3-Phosphoglycerate
2-Phosphoglycerate
Phosphoenolpyruvate
Lactate
LACTATE
Pyruvate
Cytoplasm
Acetyl-CoA
Mitochondria
*Stars indicate key controlling enzymes and steps of glycolysis. The light-color star connecting pyruvate to acetyl CoA is
not a glycolytic step. It marks the pyruvate dehydrogenase enzyme (PDH) that is required for aerobic respiration of
glucose.



Glucose 6-Phosphate
Glycogen phosphorylase catalyzes conversion of
glycogen to glucose 1-phosphate.
Hexokinase and/or glucokinase catalyze
conversion of glucose to glucose 6-phosphate.
The two glucose phosphates—glucose 1phosphate and glucose 6-phosphate, are readily
interchangeable—using phosphoglucomutase as
their catalyst.

Inhibited by the product (G-6-P); hence, it works
better if products are utilized quickly (e.g. fasting
conditions).
Glycogen
Glucose
Hexokinase
Phosphorylase
G-6-Phosphatase
Phosphoglucomutase





Hexokinase (HK)
Most tissues have hexokinase
Broad Specificity (acts on most sugars).
Affinity for glucose is high (low Km); it works
maximally during fasting glucose conditions.
Helps to provide G6P to tissues during low blood
glucose levels.
Low Vmax (cannot phosphorylate high glucose
quantities). It has low capacity for working on
glucose.
Glucose 1-P
Glucose must first be converted to
glucose 6-P before descending
down the glycolytic pathway.
38. Northwestern Medical Review, Bare Minimum Review Series, 2012
Glucose 6-P
Glycolytic Pathway
Note: Normally about 10-20% of red cell glycolytic
pathways are shunted toward 2,3 DPG formation.
This is an important shunt in the red cells and it is
normally stimulated on the tissue sides as a result of
accumulation of metabolic byproducts such as CO2
and hydrogen ions that drop the pH.
Glucose
Hexokinase
Glucose 6-P
Glyceraldehyde 3-P
Note: Addition of phosphate to hexoses would trap glucose
within the cells. It also energizes the molecule in
preparation for further enzymatic activities. As we will see
later in the absence of alternative pathways to utilize the
phosphorylated sugars, this trapping will increase the
osmotic pressure and lyses or damages the cells.





Glucokinase
Glucokinase is the unique liver brand of
hexokinase.
Shows affinity for glucose when glucose level is
very high (high Km).
Handles huge incoming venous portal glucose
after meals (high vmax). It has high capacity for
working on glucose.
Synthesis is stimulated by insulin.
As opposed to hexokinase, it is not inhibited by
glucose 6-phophate.
Note: The only other cells in the body with
glucokinase are the pancreatic beta-cells.
1. Why do people not get hyperglycemic after
carbohydrate meals?
______________________________________
______________________________________





Hexokinase (HK) Deficiency
Autosomal recessive disease
Hemolytic anemia
Decreased formation of 1, 3 DPG (1,3
diphosphoglycerate).
1,3 DPG is normally converted to 2,3 DPG by a
mutase in the red cells.
Hence, in hexokinase deficiency the 2,3 DPG
level is decreased.
Hexokinase deficiency is associated
with downstream deficiency of 1,3 DPG
and as a result a deficiency of 2,3 DPG
in the red cells!
1,3 Diphosphoglycerate
3 Phosphoglycerate
Mutase
Red Cells
Phosphatase
O2 + Hb
2,3 Diphosphoglycerate
HbO2
Pyruvate





2,3 Diphosphoglycerate
Most abundant organic phosphate source of
RBC
Regulates binding of oxygen to hemoglobin
If absent, Hb (hemoglobin) shows a high affinity
for oxygen; if present, Hb unloads oxygen.
HbO2 (oxyhemoglobin) + 2,3DPG  Hb2
(deoxyhemoglobin) + O2 + 3PG
2,3DPG level in RBC increases in chronic
hypoxia, COPD, asthma, and high altitude.
Note: It is postulated that acidity of blood at the
tissue level redirects part of red blood cell glycolytic
pathways toward 2,3 DPG formation via activating a
mutase that converts 1,3 DPG to 2,3 DPG. As a
result red cells unload their oxygen to the tissues and
become deoxygenated. Deoxyhemoglobin, on the
other hand, is a perfect buffer and it buffers the acids.
As a result of drop in the acidity a phosphatase
enzyme gets activated and converts 2,3 DPG to 3phopshoglycerate (i.e. back to the main glycolytic
pathway). Also note that in the lung vasculature due
to lower levels of 2,3 DPG and less acidity of the
blood hemoglobin can more aggressively load
oxygen. The same mechanism also explains why in
COPD and asthma (which are associated with
respiratory acidosis and high CO2 levels) there is a
higher level of 2,3 DPG in red cells.
39. Northwestern Medical Review, Bare Minimum Review Series, 2012


Findings in Hexokinase (HK) Deficiency
Low 2,3-DPG of the red cells causes high
oxygen affinity of hemoglobin for oxygen. At the
same time it makes unloading of oxygen to
tissues very difficult. This leads to left shift of the
hemoglobin oxygen dissociation curve and tissue
anoxia (or tissue hypoxia).
Lack of ATP formation due to failure of glycolytic
pathways cause hemolytic anemia.
Generalization
All glycolytic enzymes share one aspect in
common; their deficiency is associated with
hemolytic anemia!
Autosomal Recessive and Dominant Defects
Recessive
Dominant
3. Why does failure of the Na+/K+-ATPase pump
cause hemolysis of the red cells?
______________________________________
______________________________________
4. What is the only source of energy for the red
cells and why?
______________________________________
______________________________________
Findings in Hemolytic Anemia
 Premature RBC destruction
 Accumulation of hemoglobin catabolic products
(e.g. bilirubin).
 Marked erythropoiesis and reticulocytosis.

Lab Characteristics of Hemolytic Anemia
Hemoglobinemia,
hemoglobinuria
and
Hemosiderinuria (Iron overload)
Normocytic anemia (acute cases)
Jaundice (conjugated hyperbilirubinemia)
Reduced serum haptoglobin. Haptoglobin is made
by the liver and binds free hemoglobin and the
bound form is removed by the reticuloendothelial
system and spleen.
Often cause enzyme
deficiencies (e.g. hexokinase
or glucose 6 phosphate
dehydrogenase deficiencies)
Often cause structural protein
defects (e.g. collagen in
Marfan’s, or spectrin in
hereditary spherocytosis)
100% penetrance (i.e. all
patients exhibit all symptoms)
Partial penetrance (i.e. not all
patients exhibit all symptoms)
Early age of onset (e.g. infancy
to early childhood)
Variable age of onset (early
childhood through middle ages)
Both parents must be carriers
for the defective gene
One of the parents must have
the disease
5. In what form is iron absorbed from the GI?
______________________________________
Often more serious
Often less serious
6. How is iron transported in the blood?
______________________________________
2. A patient is diagnosed with hexokinase
deficiency. What is the most likely age of the
patient?
A.
B.
C.
D.
E.






7. What is the name of the storage form of iron?
_____________________________________
8. What commonly tested disease is associated
with high iron deposits in tissues and it is
characterized by diabetes, cirrhosis, and dilated
(congestive) and/or restricted cardiomyopathy?
____________________________________
8 months
8 years
16 years
32 years
50 years
Mechanism of Hemolytic Anemia
Red blood cells are 100% dependent on
anaerobic glycolysis for making their ATP.
Failure of glycolytic enzymes impedes ATP
production.
Low ATP reduces Na+/K+-ATPase activity that is
required for integrity of RBC’s membrane shape
and function. Hence: red cells swell and break
apart. Hence: Hemolytic anemia.
9. What is hemosiderin made up of?
____________________________________
10. What is normal reticulocytic index and when
does it increase or decrease?
____________________________________
____________________________________
11. What is the term used to describe diabetes as
result of hemosiderins deposit in the pancreas?
____________________________________
40. Northwestern Medical Review, Bare Minimum Review Series, 2012
12. What is the treatment for hemochromatosis?
____________________________________
____________________________________
13. Diagnosis of hemochromatosis quite often
begins by measuring the serum levels of
transferrin and ferritin. What are the two common
tests performed on the liver to diagnose
hemochromatosis?
____________________________________
____________________________________
14. What tissue of the body is most adversely
affected by glycolytic enzyme deficiencies?
____________________________________
Regulation of Phosphofructokinase
AMP & FR-2,6-BP
ATP & CITRATE
Parthenogenesis of Hemochromatosis
Excessive GI Iron
Absorption
Iron stimulates
collage formation
Cirrhosis




Iron
accumulates as
hemosiderin in
hepatocytes
Iron alters DNA
Hepatocellular
Carcinoma
Phosphofructokinase is the ratelimiting enzyme of
glycolysis
Iron induces formation
of free radicals (due to
Fenton reaction.)
Fructose 6-P
H2O2 and free radicals
cause lipid
peroxidation of cell
membranes


Necrosis of Cells
Factors Affecting Integrity of Red Blood Cells
Increased
level
of
glucose-6-phosphate
dehydrogenase (indirect positive effect)
Adequate availability of ATP
Proper activity of pentose monophosphate shunt
Maintenance of glutathione in reduced state
G-6-Phosphate: The Gateway to Other Pathways



Glucose 6-P
Glycogen
Pyruvate
6-P Gluconate
G1P

Phosphofructokinase I
Biochemical Control of PFK
The purpose of glycolysis is to produce energy
High titer of cellular AMP levels (i.e. low energy)
will activate the glycolytic enzymes in general
and PFK in particular.
High levels of intracellular ATP will shut-off the
PFK
Citrate is the first compound in the Kreb’s cycle,
and it is among the few compounds that can exit
the mitochondrial membrane and get into the
cytoplasm wherein the glycolytic enzymes are
located.
The titer of citrate increases in the mitochondria
whenever there is a high level of ATP production
by the Kreb’s cycle. Appearance of the citrate in
the cytoplasm will then convey the message of
high ATP levels to the PFK.
The next potent activator of the PFK is fructose
2,6 bisphosphate (see further below)
TCA
G6P
Glucose
Energy
Fructose 1,6-P
Ribose
Glucose 6-phosphate is the substrate of three enzymatic
pathways; glycogen synthesis and degradation (the left side
pathway in the above diagram), glycolysis (in the center) and
hexose mono-phosphate shunt (on the right).
41. Northwestern Medical Review, Bare Minimum Review Series, 2012
Fructose 2, 6-Bisphosphate
 In the Liver, F2,6-BP is formed from F6-P by
PFKII.
 F2,6-BP activates PFKI
 It inhibits F1,6-bisphosphatase, that is, the
enzyme of the reverse reaction (gluconeogenesis).
 It is an intra-cellular sensor of glucose level
 Hepatic levels are decreased with elevated
glucagon or during fasting
 Levels are increased with low glucagon and in the
fed state
 Insulin greatly increases synthesis of PFKII and as
a result stimulates the glycolytic pathways.
15. Which hormone stimulates and one inhibits PFK
II?
____________________________________
____________________________________
16. Which of the two has a higher effect in raising
output of insulin from the pancreas; a high
carbohydrate diet or IV injection of glucose?
____________________________________
____________________________________


to hemolytic anemia
Increased muscle glycogen causes myopathy,
rhabdomyelysis, increased serum CPK,
myoglobinemia and myoglobinuria.
Substrates before the step increase in muscle.
For example, fructose and glucose 6-phosphate
increase, whereas, products after the step such
as Fr 1,6-diphosphate and 1,3 DPG decrease.
17. List major causes of rhabdomyelysis:
______________________________________
______________________________________
______________________________________
______________________________________
18. What is the rate-limiting enzyme of glycolysis?
_____________________________________
Phosphofructokinase (PFK) Deficiency
Glycogen
Glucose 1-phosphate
Glucose 6-phosphate
Fructose 6-phosphate
Phosphofructokinase
PFK II
FR-6-P
Fructose 1,6-bisphosphate
FR 2,6 BP
In phosphofructokinase deficiency
glycolysis fails after the F6P step. TCA
slows down; ATP and energy levels drop.
Substrates before the step; including
glycogen, buildup in tissues.
PFK I
FR 1,6 BP
Northwestern Medical Review
Fructose 2,6 BP is made in the liver
by PFK II and it promotes glycolysis
via stimulating
Phosphofructokinase I and stops
gluconeogenesis.
Glycolysis







Last Unique Step of Glycolysis
2 Phosphoenolpyruvate
Gluconeogenesis
Phosphofructokinase (PFK) Deficiency
AKA. Type VII Glycogen Storage Disease
Inefficient muscle glucose supply causes cramps
Glucose is mainly supplied by gluconeogenesis
Increased muscle glycogen stores (glucose is
shunted toward glycogen production due to
improper glycolytic functions).
Mild Hypoglycemia. Breakdown of glycogen is
slowed down and glycogenesis is enhanced.
Rate of ATP formation (by Kreb’s Cycle)
decreases.
Red cells are deprived of ATP and there is failure
of sodium-potassium-ATPase pump. Hence,
sodium stays within the red cells and this leads
2 ADP
Pyruvate Kinase
2 ATP
2
Pyruvate
Sugars make pie!
Pyruvate (the pie!) is the end result of
carbohydrate metabolism.
Pyruvate is also a pirate because it pirates 3
carbon sugars from cytoplasm into the mitochondria.
(We will see more on this later!)
42. Northwestern Medical Review, Bare Minimum Review Series, 2012
19. A 25-year-old woman with history of heavy
menstrual bleeding complains of fatigue and
exercise intolerance. Blood analysis is significant
for hypochromic microcytic anemia. Which of the
following two iron preps would you prescribe for
her, ferric or ferrous sulfate?
_______________________________________

Formation of Pyruvic Acid
Formation of two pyruvic acid molecules from two
PEP molecules in the cytoplasm marks the end of
glycolysis. But in order for ATP (energy) to become
available pyruvate (the pie!) must be further
oxidized (eaten!) either under aerobic or anaerobic
conditions (see further below for details).

The last Step of Glycolysis

PEP
P
AT
PK
Mnemonic:
One of the two Ps
of PEP goes to ATP
and the other to
Pyruvate!
Pyruvate
Pyruvate kinase is the last key-controlling enzyme of glycolysis.
Generalization
The purpose of glycolysis is to make energy. As a
general rule, accumulation of downstream products
and abundance of sources of cellular energy such as
ATP and NADH inhibit the glycolytic pathway.
Conversely, abundance of uphill substrates (that is,
the preceding compounds) promotes the pathway.
As we will see later, this principle also holds for the Kreb’s cycle.
Regulation of Pyruvate Kinase
FR-1,6-BP
Phosphoenol
Pyruvate
ATP & ALANINE
Pyruvate Kinase
Pyruvate
Pyruvate Kinase is the Last Enzyme of Glycolysis





Pyruvate Kinase (PK) Deficiency
Normal RBC lacks mitochondria, and it is fully
dependent on anaerobic glycolysis for ATP.
Deficient individuals have a low Pyruvate Kinase
level in their red blood cells
Hence, the patients have low RBC glycolysis,
and low ATP production. Low ATP affects
K+/Na+-ATPase pump, and leads to membrane
damage, lysis and hemolytic anemia.
Consequence is anemia, marked increase in
reticulocytic number and mild jaundice.
Failure of the last step of glycolysis
Abnormal
accumulation
of
glycolytic
intermediates before the step
High 2,3-DPG production and abnormally low
oxygen affinity of Hb (right shift of the Hb-oxygen
dissociation curve)
Treatment Options: Oxygen therapy, blood
transfusion (serious cases), folate or B12
administration and splenectomy.
Note: Spleen gets sequestered with and lyses the PK
deficient red cells. As such removal of spleen would
help to minimize the rate of hemolysis. Furthermore,
due to high levels of red cell breakdown these
patients are at risk for splenomegaly and rupture of
spleen.
Hexokinase
Contrast of Hexokinase and
Pyruvate Kinase Deficiencies.
Glycolytic Pathway
Pyruvate Kinase
Hexokinase stops glycolysis at the inlet and pyruvate
kinase at the outlet. Both of them slow down the TCA
function and energy-producing machinery. In the case of
hexokinase nothing past the step, including 2,3 DPG, is
produced. In the case of pyruvate kinase, every thing prior
to the step, including 2,3 DPG accumulates in the cell.
TCA
Pyruvate Kinase and Hexokinase via modulation of
the 2,3 DPG levels within the red cells affect
oxygenation of hemoglobin. Pyruvate Kinase
deficiency causes increased 1,3 DPG levels in the
red cells and as such it resembles hypoxemic
conditions. Hence, in this deficiency unloading of
oxygen to the tissues is enhanced.
20. Where in the cell does the glycolytic pathway
take place?
_____________________________________
43. Northwestern Medical Review, Bare Minimum Review Series, 2012
Summary of Glycolysis
A 4-year-girl is being evaluated for anemia and jaundice.
Lab results are significant for increased reticulocytes,
reduced serum haptoglobin, and increased urinary
hemosiderins. No sickle cells or spherocytes are
identifiable in the child’s blood. She has mild
splenomegaly and lacks hepatomegaly. There is no
history of any recurrent infections.
22. What is the best descriptive term for explaining
above finding?
_______________________________________
23. What are your first impression top differentials?
______________________________________
______________________________________
______________________________________
Right shift of hemoglobin (Hb) dissociation curve increases P50
(partial pressure of oxygen that saturates 50% of available Hb
sites). That is for any PO2 level saturation of Hb will decrease.
Associations of Hemoglobin-Dissociation Curve
Left Shift
Right Shift
 Low 2,3 DPG in RBC
 High hemoglobin oxygen
affinity
 Low unloading of O2
 Carbon Monoxide
 Methemoglobinemia
 Fetal hemoglobin
 Hexokinase and
phosphofructokinase
Deficiencies
 High 2,3 DPG in RBC
 Low hemoglobin oxygen
affinity
 High unloading of O2
 Increased acidity
 Increased PCO2
 High Temperature
 Chronic hypoxia
 High Altitude
 COPD/Lung Disease
 Pyruvate Kinase
Deficiency
21. Both carbon monoxide poisoning and
methemoglobinemia cause left shift of
hemoglobin-oxygen dissociation curve and make
unloading of oxygen more difficult. Why oxygen
therapy is still the mainstay therapy in both
conditions?
_______________________________________
_______________________________________
24. What are your second impression top
differentials?
______________________________________
______________________________________
______________________________________
25. What makes above enzyme deficiencies similar?
______________________________________
______________________________________
______________________________________
26. Which of the three is the most likely cause of the
above findings and also the most commonly
tested one on the COMLEX and USMLE exams?
______________________________________
27. What is by far the most common cause of
hemolytic anemia as a result of an enzyme
deficiency?
______________________________________
28. What unique clinical finding will set PFK apart
from the other two glycolytic deficiencies?
______________________________________
29. In which of the aforementioned glycolytic
deficiencies the level of 2,3DPG increases in the
red cells and the patient assume a right shift of
the hemoglobin dissociation curve?
______________________________________
44. Northwestern Medical Review, Bare Minimum Review Series, 2012
31. Which of the patients in the above three
glycolytic enzyme deficiencies will be more
benefitted
from
oxygen
therapy?
______________________________________
30. Why patient with hexokinase deficiency present
with the most profound hemolytic anemia
among the three glycolytic deficiencies?
______________________________________
Summary of Glycolytic Enzyme Deficiencies
HK
PFK
PK
All are major glycolytic enzyme deficiencies
All are rare autosomal recessive defects
Hemolytic anemia
Treatment Options: Blood transfusion, splenectomy, and foliate and vitamin B12
administration
 Most fatal hemolytic
 Glycogen storage
 The most common glycolytic
anemia among common
disease VII
enzyme deficiency
glycolytic enzyme
 Rhabdomyelysis,
deficiencies
myoglubinemia and high  Right shift of Hb-oxygen
dissociation curve and
 Left Shift
serum CK
difficulty with loading oxygen
 Treated with blood
 Left Shift of Hb-oxygen
on hemoglobin
transfusion
dissociation curve
1.
2.
3.
4.
 Oxygen therapy
32. What is the major source of energy for the red
cells during serious starvation?
______________________________________
33. What is the homeostatic value of maintaining
blood glucose levels at about 90 mg/dl?
______________________________________
34. You are looking at the blood smear of a 6-yearold child with hemolytic anemia due to pyruvate
kinase deficiency. What type of anemia based
on the size of red cells do you expect to see?
______________________________________
35. What two clinical conditions are common
causes of the same type of anemia based on
the size of the red cells?
______________________________________
______________________________________
45. Northwestern Medical Review, Bare Minimum Review Series, 2012
ANSWERS
1.
Under the influence of insulin, glucokinase traps
(sequesters) glucose in the hepatocytes and convert
them to glycogen.
2.
Autosomal recessive genetic conditions often are
diagnosed at early age. Therefore the best answer is
[A].
3.
The pump extrudes sodium and returns potassium
back into the red cells. In the absence of proper pump
function sodium is retained in the red cells and
increases osmotic pressure. This leads to swelling and
early destruction of the cells.
4.
Mature red cells do not have mitochondria. Therefore
they are not capable of performing aerobic respiration.
For this reason they cannot use fatty acids and ketone
bodies as a source of energy. The only energy
producing mechanism available to the red cells is
anaerobic glycolysis that yields two ATP molecules for
every glucose molecule that they consume.
5.
Iron is absorbed in ferrous form (Fe++)
6.
Transported by transferrin that is a protein made by
the liver in the ferric form (Fe++)
7.
The storage form of iron is known as ferritin. It is a
protein that combines with iron.
8.
Hemochromatosis! The genetic form of the disease is
associated with excessive absorption of iron from gut.
It is due to mutation in the HFE gene. It is more
common among the Caucasians and in particular Irish
population.
9.
Hemosiderins are made up of iron, protein and
polysaccharides. They accumulate in various tissues
including heart, pancreas, liver and GI mucosa and
often cause death of the tissues.
10. Reticulocytes are immature red cells. Typically about
1% of the red cells are reticulocytes. Reticulocytes
develop and mature in the red bone marrow and then
circulate for about a day in the blood stream before
developing into mature red blood cells. Like mature
red blood cells, reticulocytes do not have a cell
nucleus. They are called reticulocytes because of a
reticular (mesh-like) network of ribosomal RNA that
becomes visible under a microscope with certain
stains such as new methylene blue. In hemolytic
conditions the reticulocytic index is more than 1. In
contrast in bone marrow diseases it may drop to
values less than 1.
11. Bronze diabetes
12. Periodic phlebotomy and oral administration of zinc.
Zinc competes with iron for absorption from GI.
13. Liver is one the main organs for accumulation of iron
deposits. Ferri-Scan is a MRI-based test to noninvasively and accurately measure liver iron
concentrations. It identifies iron deposits in the
hepatocytes. Currently it is a preferable choice over
the widely used liver biopsies (and staining of the
tissue with Prussian blue).
Note
that
serum
transferrin and transferrin
saturation are commonly used as screening for
hemochromatosis. Fasting transferrin saturation
values in excess of 45% is suggestive of
hemochromatosis. Transferrin saturation greater than
62% is suggestive of homozygosity for mutations in
the HFE gene. Serum ferritin provides another crude
estimate of whole body iron stores though many
conditions. But it involves a high rate of false positive
cases because inflammation, chronic alcoholism, and
fatty liver diseases also elevate serum ferritin. Normal
serum ferritin values for males are 12–300 ng/ml (and
less in females). Serum ferritin in excess of 1000
ng/ml of blood is most likely due to hemochromatosis.
14. Glycolytic enzyme deficiencies most adversely affect
the red cells as they can only utilize glucose as the
(sole) source of energy.
15. Insulin (fed state) stimulates PFK synthesis whereas
glucagon (fasting state) inhibits it.
16. High sugar diet has more profound effect on
stimulating the insulin release because in addition to
raising sugar levels it activates parasympathetic
nervous system. Activation of parasympathetic system
greatly activates pancreatic insulin release.
17. Causes of rhabdomyelysis: (1) Muscle Crush (Car
accident); (2) Snake Venom; (3) Enzyme Deficiencies:
PFK deficiency, McArdle’s disease, Carnitine Acyltransferase and Carnitine deficiency, and MCAD
deficiency; (4) Drugs such as Statins, Fibric acid
derivatives, Tubocurarine plus halothane, haloperidol,
and chlorpromazine; (5) Others: Tetanus, UMN
Deficits and grand mal seizures.
18. PFK
19. Ferrous sulfate, because iron is absorbed in ferrous
form.
20. In the cytoplasm!
21. Patients who breathe hyperbaric oxygen (pressurized
100% oxygen) show improvement. Although many
scholars question the utility of the procedure. It is
speculated that by pressurized oxygen we are able to
bypass the contributions of the hemoglobin and force
feed oxygen into the mitochondria.
46. Northwestern Medical Review, Bare Minimum Review Series, 2012
22. Hemolytic anemia
23. All genetic causes of hemolytic anemia: Hexokinase
deficiency, Phosphofructokinase deficiency, Pyruvate
kinase
deficiency,
Glucose
6
phosphate
dehydrogenase deficiency, Hereditary Spherocytosis,
Sickle Cell disease, Beta-thalassemia (also alphathalassemia, and Hereditary elliptocytosis
24. Hexokinase
deficiency,
Phosphofructokinase
deficiency, and Pyruvate kinase deficiency
25. They are key glycolytic enzyme deficiencies. They all
autosomal recessive. All three deficiencies cause
hemolytic anemia!
26. Hexokinase deficiency
27. G6PD deficiency
28. Rhabdomyelysis as a result of glycogen build-up in
the cytoplasm of striated muscle that leads to
myoglobinuria, high serum CK and creatine.
29. PK deficiency. The other two are associated with low
2,3DPG and a left shift
30. Hexokinase is also required for HMP shunt function
in the red cells. The shunt makes NADPH that safeguards red cell membrane against oxidizing agents
via increasing the level of reduced glutathione. So
these patients in addition to osmotic damage of the
red cells that is shared among all three are at further
risk of oxidizing damages to red cell membrane. Note
that the red cells due to 100% reliance on glucose
can only use hexose monophosphate shunt and
anaerobic glycolysis as the source of energy.
31. Patients with PK deficiency as a result of right shift of
the oxygen dissociation curve.
32. Glucose. They cannot utilize any other food source.
33. Even though the brain cells under normal conditions
and early starvation rely on glucose, the major
contribution of maintain glucose is to nourish red
cells and promote respiration that is vital to survival.
34. Normocytic anemia. Hemolytic anemias commonly
cause normocytic anemia
35. Kidney diseases due to lack of erythropoietin and
hemorrhage cause normocytic anemia.
47. Northwestern Medical Review, Bare Minimum Review Series, 2012
Post-Test
Total Items: 50
Allotted Time: 60 Minutes
Minimum Passing Score: 33 correct answers
* The passing score is calibrated to reflect performance on the Step 1 examination
48. Northwestern Medical Review, Sample Review Book Series, 2012
POST-TEST
1. You are evaluating a 20-year-old female patient
with a 4-year history of renal disease for complaint
of visual problems. History shows that her disease
has begun with an abrupt incidence of hematuria
that has gradually progressed to proteinuria of
more than 3.5 grams per day. A recent renal
biopsy of the patient has been significant for
presence of dense homogenous deposition along
the glomerular basement membrane and in the
mesangium.
Which of the following mechanisms will better
explain the etiology of this patient’s renal findings?
A.
B.
C.
D.
E.
Activation of classic complement pathway
Activation of alternate complement pathway
Activation of lectin complement pathway
Circulating antistreptolysin O antibodies
Circulating HLA-B27 antibodies
C. Ankylosing spondylitis is a recessive X-linked
condition
D. Estrogen in females inhibits expression of the
disease
E. Genes for the disease are located on Y
chromosome
5. Celiac disease is associated with which of the
following conditions?
A.
B.
C.
D.
E.
Henoch-Schonlein Purpura
Systemic lupus erythematosus
Dermatitis herpetiformis
Ankylosing spondylitis
Rheumatoid arthritis
6. Which of the following immunofluorescence
patterns of glomerular deposits is seen in patients
with rapidly progressive glomerulonephritis?
2. Which of the following renal diseases is classified
as a nephrotic syndrome?
A.
B.
C.
D.
E.
Goodpasture’s syndrome
Henoch-Schonlein purpura
Membranous glomerulonephritis
Membranoproliferative glomerulonephritis
Alport’s syndrome
3. Which of the following findings will distinguish
membranoproliferative glomerulonephritis (MPGN)
type I from type 2?
A.
B.
C.
D.
E.
Red cell casts in urine
Activation of complements in the glomeruli
Increased number of mesangial cells
Mesangial immune deposits
Despite therapy may progress to end-stage
renal failure
4. Patients with ankylosing spondylitis are often
young males that in addition to chronic
inflammatory disease of axial skeleton at times are
also presented with mesangial immune deposits.
Which of the following statements is a better
explanation for the fact that ankylosing spondylitis
is more common in males?
A. Testosterone potentiates effects of the HLAB27 antibodies on the joints and tissues.
B. Testosterone potentiates effects of the IgA
antibodies on the joints and tissues.
7. You are examining a 3-week female infant who is
admitted to the hospital for the complaint of
extreme temperature fluctuations, diarrhea,
vomiting and apnea of 24-hour duration. On
examination the baby is lethargic, bradycardic,
hypotensive, and extremely jittery. Head
examination is significant for bulging of fontanelles.
Based on your clinical suspicion you will order and
immediate blood analysis and spinal tap and also
start the baby on I.V. ampicillin 100 mg/kg q12hr
and gentamicin 2.5 mg/kg q12hr.
Results of the spinal tap indicate elevated CSF
pressure; leukocytosis with 90% PMNs; glucose,
35 mg/dl; and protein, 400 mg/dl. CSF gram
staining is significant for gram-positive bacteria
with coccoid morphology. Which of the following
laboratory tests would be the most helpful in
identifying the cause of the cerebrospinal findings
in the patient?
49. Northwestern Medical Review, Sample Review Book Series, 2012
A.
B.
C.
D.
E.
Coagulase test
Catalase test
India ink staining
Camp test
Optochin-sensitivity
Use this Case for Questions 8 and 9
A 21-year-old female college student is brought to the
emergency department for evaluation of fever, vomiting
and severe myalgia that she has had for about 7 hours.
On admission she is mildly confused and has high
temperature and high pulse rate. She is in her period
and uses tampons. Her systolic pressure is 70 and
diastolic 30. She has a rash resembling sunburn,
particularly on her palms and soles. History is
significant for heavy menstrual periods.
8. Assuming an infectious cause for her symptoms,
which of the following ionic conditions would have
facilitated the pathogenesis of her condition?
A.
B.
C.
D.
E.
Hypocalcemia
Hypomagnesemia
Hyponatremia
Hypochloremia
Hypokalemia
9. The patient immediately receives an antibiotic
treatment with nafcillin. But her symptoms not only
persevere but they get augmented within the next
36 hours. She is then scheduled to receive an
alternative IV medication. As part of her therapeutic
management her serum creatinine level is
monitored quite carefully. Which of the following is
the alternative IV medication?
A.
B.
C.
D.
E.
Trimethoprim-sulfamethoxazole
Methicillin
Vancomycin
Dicloxacillin
Clindamycin
______________________________
10. Seven adults are admitted to the emergency room.
All of them are nauseating and presented with
common symptoms that include vomiting, flushing,
abdominal cramps and diarrhea. They all claim that
they have been to the same buffet-type restaurant
and their symptoms began while they have been
still dinning at the restaurant. History shows that all
of them have consumed a few hot fish dishes that
included anchovy, herring, and mahi-mahi.
Assuming that the patients are resented with
enterotoxicity, which of the following is the most
probable cause of these findings?
A.
B.
C.
D.
E.
Staphylococcus aureus
Bacillus cereus
Scombroid
Vibrio parahaemolyticus
Enterotoxigenic E. coli
11. X-ray of the hip of an 8-year-old boy with history of
recurrent bone pain is significant for avascular
necrosis of the head of femur. If the child has been
diagnosed with osteomyelitis on several occasions
in the past, which of the following conditions would
be the underlining cause of his avascular necrosis?
A.
B.
C.
D.
E.
Sickle cell disease
Cystic fibrosis
Vascular thrombosis
Scalded Skin Syndrome
Legg-Calvé-Perthes syndrome
12. You are giving a lecture in microbiology to medical
students. The topic of your lecture is food
poisoning. A student in the audience is interested
to know which of the following food poisonings as a
result of preformed toxin is sensitive to heating of
the food; what would be your answer?
A.
B.
C.
D.
E.
Staphylococcus aureus
Bacillus cereus
Scombroid
Botulinum
Salmonella typhi
13. Which of the following bacteria is the most
common cause of necrotizing fasciitis?
A.
B.
C.
D.
E.
Staph auresus
Strep pyogenes
Pseudomonas aeroginosa
Clostridium perfringens
Bacillus anthrax
50. Northwestern Medical Review, Sample Review Book Series, 2012
14. In treatment of infections with Staph aureus
nafcillin is preferable over methicillin, why?
A. Nafcillin in contrast to methicillin is resistant to
beta-lactamase
B. Nafcillin has more potency for destroying the
cell wall of gram-positive bacteria
C. Nafcillin is effective against methicillinresistant staph aureus (MRSA)
D. Methicillin causes serious nephrotoxicity
E. Methicillin causes serious neurotoxicity
15. A 25-year-old man is admitted to the emergency
room about 10 days ago for urinary incontinence
due to urinary stones, and also for an infected skin
wound. He was catheterized with Foley catheter for
his incontinence and received topical care for his
skin infection. 8 days later he is admitted to the
ICU due to sepsis. All of the following listed
bacteria might have caused his sepsis as a result
of skin infections and/or contamination EXCEPT:
A.
B.
C.
D.
E.
Staph aureus
Streptococcus pyogenes
Pseudomonas aeroginosa
Clostridium tetani
Klebsiella pneumoniae
16. A patient with recent history of prosthetic
valvuloplasty is presented with sepsis due to
vancomycin resistant Staph epidermidis. Which of
the following options is a reasonable back-up
medication?
A.
B.
C.
D.
E.
Ceftriaxone
Aztreonam
Trimethoprim-sulfamethoxazole
Imipenem
Gentamicin
17. A 17-year-old girl is being admitted to the OB/GY
clinic for the complaint of amenorrhea. Examination
reveals lack of development of secondary sex
characteristics. Blood pressure is 180/110; result of
funduscopy
is
normal;
she
has
no
lymphadenopathy or hepatosplenomegaly. Blood
analysis indicates hypokalemia, hypernatremia,
metabolic alkalosis, and suppressed renin. Urinary
analysis confirms a high level of urinary
gonadotropins. Which of the following deficiencies
is the most likely cause of this patient’s problems?
(A)
(B)
(C)
(D)
(E)
11-hydroxylase
21-hydroxylase
17-hydroxylase
Desmolase
Hydroxysteroid dehydrogenase
18. Your patient is a 12-year-old male patient with
history of precocious puberty and congenital
adrenal hyperplasia. On examination he has a
blood pressure of 90/60. Which of the following
enzyme deficiencies is the most likely cause of his
condition?
(A)
(B)
(C)
(D)
(E)
11-hydroxylase
21-hydroxylase
17-hydroxylase
Desmolase
Hydroxysteroid dehydrogenase
19. Which of the following conditions is associated with
eosinophilia?
A.
B.
C.
D.
E.
Addison’s disease
Cushing’s disease
Congenital adrenal hyperplasia
Candidiasis
Paraneoplastic Cushing’s due to oat cell
carcinoma
20. A 35-year-old female patient with estrogensensitive breast cancer will be more effectively
treated with which of the following cited
medications?
A.
B.
C.
D.
E.
Aminogluthemide
Spironolactone
Beclomethasone
Flutamide
Human menopausal gonadotropins
51. Northwestern Medical Review, Sample Review Book Series, 2012
21. Titer of Corticotropin-releasing hormone (CRH)
from hypothalamus is increased in which of the
following conditions?
A.
B.
C.
D.
E.
Oat cell carcinoma of the lung
Cushing’s disease
Iatrogenic hypercortisolism
Unilateral adrenocortical adenoma
Water-house Friderichsen syndrome
22. Sensitivity to a very high dose of dexamethasone
suppression is indicative of which of the following
conditions?
A.
B.
C.
D.
E.
Adrenal carcinoma-induced hypercortisolism
Oat-cell carcinoma-induced hypercortisolism
Pituitary adenoma-induced hypercortisolism
Drug-induced (iatrogenic) hypercortisolism
Adrenal adenoma-induced hypercortisolism
23. A 30 year-old woman complains of gradual weight
gain. She has normal menstrual periods.
Examination shows that she has a blood pressure
of 180/110 mm Hg. Her serum creatinine, sodium,
potassium, and chloride are within normal ranges.
Further lab findings indicate hypocapnia and
hyperglycemia. Which of the following radiological
findings would you expect in this patient?
A.
B.
C.
D.
E.
Thyroid mass
Ovarian cysts
Nodules within one of her lungs
Mass in one of her adrenals
Pituitary tumors
24. In humans, surgical procedures that involve total
hypophysectomy are less fatal than total
adrenalectomy without hormone replacement
therapy. A reasonable explanation for this striking
fatal effect would be which of the following?
A. Hypophysectomy does not affect aldosterone
release from the adrenal cortex
B. After hypophysectomy, the adrenal cortex
undergoes total atrophy due to lack of ACTH
C. Levels of estrogen and testosterone do not
drop after hypophysectomy
D. Hypophysectomy does not affect the cortical
zones of the adrenals
E. The pituitary gland plays only a small role in
maintaining the adrenal's integrity
25. A 60 year-old-woman is scheduled for an
emergency appendectomy. Her history shows that
18 months ago she was admitted to the hospital as
a result of severe headaches, joint pain,
tenderness of her temporal areas, and sudden loss
of visual acuity in her left eye. Her erythrocytic
sedimentation rate (ESR) at the time of admission
was elevated at 87 mm/h. At the time, a
presumptive diagnosis of giant cell arteritis was
made, and she was started on a regimen of
prednisone, 60 mg/d, with dramatic improvement in
her symptoms. The prednisone dosage was then
gradually tapered off as her serum ESR level
approached the normal level. Upon admission for
appendectomy, she claims that she took her last
daily dosage of prednisone 5 days ago.
Which of the following medications would you
consider giving to this patient prior to her
appendectomy?
A.
B.
C.
D.
E.
ACTH
Fludrocortisone
Indomethacin
Vasopressin
Prednisone
26. Which of the following findings is the earliest
detectable finding associated with the use of
inhaled corticosteroids?
A.
B.
C.
D.
E.
Adrenal atrophy
Localized Candidiasis
Osteoporosis
Peptic ulceration
Weight gain
27. Your patient is a 65 year-old woman with
symptoms of congestive heart failure who has had
several episodes of syncope within the past 4
months. She claims that she had no significant
cardiovascular problems up until 6 months ago
when she started experiencing chest pain during
exercise. Auscultation of the carotid pulse indicates
that the pulse is weak and late relative to the heart
contraction. Which of the following findings would
you also expect to see in this patient?
A. Discrepancy of pressure between the two
brachial arteries
B. Split of second sound that disappears with
inspiration
C. Systolic murmur
D. Pulsus paradoxus
E. All of the above
52. Northwestern Medical Review, Sample Review Book Series, 2012
28. Which of the following conditions is presented with
systolic murmur?
A. Aortic insufficiency, mitral stenosis and
ventricular septal defect
B. Aortic stenosis and aortic insufficiency
C. Aortic and pulmonary stenosis, and mitral
insufficiency
D. Aortic and pulmonary stenosis, and mitral
insufficiency
E. Aortic and pulmonary stenosis, and mitral
insufficiency
29. Which of the following patients may most likely
present with paradoxical split of the second heart
sound?
A.
B.
C.
D.
E.
A patient with systemic hypertension
A patient with pulmonic valve stenosis
A patient with Eisenmenger’s syndrome
A patient with right bundle branch block
A patient who is taking dobutamine
30. Compared to normal heart what will happen to endsystolic ventricular volume in aortic stenosis?
A.
B.
C.
D.
Increases
Decreases
It is not affected
It increases with inspiration and decreases
with expiration
E. It increases with expiration and decreases with
inspiration
31. A 72-year-old woman who has been diagnosed
with aortic stenosis is seen for the complaint of
frequent episodes of angina. Which of the following
options is the most important cause of angina in
the patient?
A. Rise of intra-ventricular pressure during
systole
B. Concentric hypertrophy of the left ventricle
C. Low arterial systolic pressure
D. Low arterial diastolic pressure
E. Increased activity of renin-angiotensin system
32. Why patients who are diagnosed with aortic
stenosis at earlier ages are often males?
A. Tricuspid valves have a higher tendency for
calcification at earlier age
B. High titers of estrogen masks the symptoms of
aortic stenosis in younger women
C. Rheumatic heart disease is more common in
males
D. Inheritance of bicuspid valves has a sex-linked
pattern
E. Older women visit their doctors more than the
younger women.
33. A 17 year-old boy is seen by the ophthalmologist
for evaluation of visual problems. The height of the
patient is 8 feet 4 inches. Which of the following
visual field defects is most likely exemplified by the
patient?
A.
B.
C.
D.
E.
Complete blindness in one eye
Binasal heteronymous hemianopia
Bitemporal heteronymous hemianopia
Bilateral scotoma
Homonymous hemianopia
34. Also called “Blue Cell Tumor”, this brain tumor
commonly occurs in children. The most common
location for it is the midline of cerebellum. It may
spread into the CSF. What is this tumor?
A.
B.
C.
D.
E.
Medulloblastoma
Neuroblastoma
Ganglioneuroma
Meningioma
Astrocytoma
35. This picture is the left eye report of a 25year-old female patient during her
ophthalmologic examination. She is using
oral contraceptives. Which of the
following conditions is the most likely
underlying cause of this finding?
A. Thrombosis of left posterior cerebral artery
B. Thrombosis of basilar artery at the junction of
union of two vertebral arteries
C. End-stage glaucoma
D. Multiple sclerosis
E. A tumor of the occipital lobe
53. Northwestern Medical Review, Sample Review Book Series, 2012
36. This picture is the report of a
patient to his ophthalmologist
during his eye examination.
Which of the following options
is the correct diagnosis for this
finding?
A. Right inferior quadrantanopia and left superior
quadrantanopia.
B. Right superior quadrantanopia and left inferior
quadrantanopia.
C. Bitemporal heteronymous quadrantanopia and
binasal heteronymous quadrantanopia
D. Inferior heteronymous and superior
homonymous quadrantanopia
E. This type of visual field dysfunctions cannot be
demonstrated in humans
37. A patient has been
diagnosed with metastatic
brain tumors. The tumors
have severed his visual
pathways as it is shown in
the diagram. Which of the
following visual field defects
are expected in the patient?
A.
B.
C.
D.
Right homonymous hemianopia
Superior homonymous hemianopia
Inferior homonymous hemianopia
Right inferior quadrantanopia and left superior
quadrantanopia
E. Bilateral macular sparing
38. Following diagram show severance of projections
passing through the temporal lobe. What type of
visual field defect is caused by this lesion?
39. A three-year-old child has an autosomal recessive
disease that causes hemolysis of his red blood
cells. It is determined that his disease is due to an
enzymatic deficiency that affects his hexose
monophosphate shunt in addition to his glycolytic
pathways. Which of the following diseases has a
blood smear and red blood cell distribution
width (RDW) finding that is most closely similar to
this patient?
A.
B.
C.
D.
E.
40. Which of the following cited conditions decreases
the affinity of hemoglobin for oxygen?
A. Methemoglobinemia
B. Increased 2,3 diphosphoglycerate in the red
cells
C. Carbon monoxide poisoning
D. Phosphofructokinase deficiency
E. Hexokinase deficiency
41. Which of the following enzymatic deficiencies is by
far the most common cause of red cell hemolysis?
A.
B.
C.
D.
Hexokinase deficiency
Phosphofructokinase deficiency
Pyruvate kinase deficiency
Glucose 6 phosphate dehydrogenase
deficiency
E. Phosphoglucomutase deficiency
42. Which of the following conditions is most likely
associated with high titer of fructose 2, 6
bisphosphate levels in the hepatocytes?
A.
B.
C.
D.
E.
A.
B.
C.
D.
E.
Kidney disease
Liver disease
Orotic aciduria
Thalassemia
Anemia of chronic disease
Hexokinase deficiency
Increased parasympathetic tone
Uncontrolled type I diabetes
Pheochromocytoma
Glucagonoma
Right inferior quadrantanopia
Left inferior quadrantanopia
Right superior quadrantanopia
Left superior quadrantanopia
Right heteronymous hemianopia
54. Northwestern Medical Review, Sample Review Book Series, 2012
Use this Case for Question 43-45
A 12-year-old boy is seen for the complaint of muscle
pain and exercise intolerance. History shows that he
better tolerates aerobic exercises such as walking and
cycling, especially after approximately the first 10 to 15
minutes of exercise. Prior clinical studies have shown
that the patient’s cramps were electrically silent and his
venous lactate levels failed to increase upon ischemic
exercises. Urinalysis of the patient is significant for
myoglobinuria. Further history indicates that his older
sister has also been presented with similar findings
during her early childhood.
43. Assuming that the patient also has hemolytic
anemia, which of the following options will be the
most likely cause of these findings?
A.
B.
C.
D.
E.
McArdle’s Disease
Pompe’s disease
Phosphofructokinase deficiency
Pyruvate dehydrogenase deficiency
Carnitine-acyltransferase deficiency
44. If the patient does not have red blood cell
hemolysis but the muscle biopsy shows increased
concentrations of glycogen, which of the following
options will be the most likely cause of these
findings?
A.
B.
C.
D.
E.
McArdle’s Disease
Pompe’s disease
Phosphofructokinase deficiency
Pyruvate dehydrogenase deficiency
Carnitine-acyltransferase deficiency
45. If the patient does not have red blood cell
hemolysis but the muscle biopsy shows increased
concentrations of glycogen, which of the following
options is the most adverse long-term effect of this
patient’s disease?
A. Paralysis of extremity muscles
B. Heart failure
C. Pancreatic failure
D. Liver failure
E. Renal failure
___________________________________________
46. In glycolysis, the only substrate level
phosphorylation occurs in a reaction catalyzed by
the enzyme:
A.
B.
C.
D.
E.
glucose-6-phosphatase
pyruvate dehydrogenase
pyruvate kinase
succinate thiolase
citrate lyase c
47. Which of the following enzymes is a gluconeogenic
enzyme that reverses the last glycolytic step?
A.
B.
C.
D.
E.
Phosphoenolpyruvate carboxylases
Pyruvate dehydrogenase
Pyruvate kinase
Glucose 6 phosphatase
Glucose 6 phosphate dehydrogenase
48. Which of the following hemolytic conditions is an
autosomal dominant condition?
A. Sickle cell disease
B. Beta-thalassemia
C. Glucose 6 phosphate dehydrogenase
deficiency
D. Hereditary spherocytosis
E. Glucose 6 phosphate dehydrogenase
deficiency
49. A patient is presented with dyspnea, fatigue and
exercise intolerance. He has normal heart and lung
functions and a hematocrit of 25%. Which of the
following tests is the most informative next step in
the diagnosis of his condition?
A.
B.
C.
D.
E.
Coomb’s test
Red cell enzyme studies
Mean corpuscular volume evaluation
Electrophoresis of hemoglobin
Measurement of the level 2,3
diphosphoglycerate in the red cells
50. What is the most fatal consequence of serious and
untreated anemia?
A.
B.
C.
D.
E.
Myocardial infarction
Liver failure
Respiratory failure
Stroke
Kidney failure
55. Northwestern Medical Review, Sample Review Book Series, 2012
Answers
1.
[B]. The patient has type II MPGN. The etiology of the
disease is most likely due to uncontrolled activation of
the alternate complete system.
2.
[C]. Membranous glomerulonephritis is a nephrotic
syndrome. It should not be mistaken with
membranoproliferative glomerulonephritis (MPGN) that
has a nephritic pattern.
3.
[D]. Only MPGN type I is presented with mesangial
and/or sub-endothelial deposits. All other cited options
are shared between Type I and II.
4.
[A]. Ankylosing spondylitis can affect both men and
women. It is postulated that high testosterone levels in
young adult males may have positive correlations with
damaging expression of HLA-B27 deposits in the axial
joints and nephrons.
5.
[C]. Dermatitis herpetiformis has a strong association
with celiac disease. Note that all the cited conditions are
associated with IgA nephropathy and mesangial IgA
deposits.
6.
[E]. Rapidly progressive glomerulonephritis is also
known as crescentic nephritis. Typical glomerular
pattern quite often presents with crescents of fibrin
deposits within the Bowmen’s capsule.
7.
[B]. The baby has symptoms of sepsis and meningitis.
The top three causes of meningitis in neonates are
Streptococcus agalactiae, Listeria and E. coli. Only
streptococcus and listeria are gram-positive. Listeria is
catalase-positive and Streptococcus, negative. Note that
listeria has coccobacillary morphology and as such its
light microscopic distinction from Streptococcus
agalactiae is somehow difficult. Note that due lack of full
immunologic potency the neonates may not present with
classic symptoms of meningitis like nuchal rigidity. As
such early signs are for most part nonspecific and subtle
and do not differentiate between fungi, bacteria or
viruses. The most common symptoms in neonates are
diminished spontaneous activity, weak sucking, apnea,
bradycardia, and temperature instability. Also note that
coagulase test would be helpful to distinguish staph
aureus (coagulase positive) from Staph epidermidis and
saprophyticus that are coagulase negative. India ink
stating is important in identifying Cryptococcus
neoformans. Positive Camp test identifies Streptococcus
agalactiae, and Optochin-sensitivity is specific for
Streptococcus pneumoniae.
8.
[B]. The patient most likely has toxic shock syndrome.
The rash on her palms and soles are most likely
scarlatiniform. Low magnesium ion concentration most
likely has caused heavy growth of toxic shock syndrome
toxin-producing staph aureus. Note that it is postulated
that tampons absorb various ions, in particular
magnesium. Low magnesium promotes heavy growth of
Staph aureus.
9.
[C]. The patient most likely has toxic shock syndrome
as a result of infection with MRSA. The number one
choice of treatment of MRSA is Vancomycin. The
common side effects of Vancomycin are flushing, shock,
ototoxicity and nephrotoxicity.
Trimethoprimsulfamethoxazole is the second choice after
Vancomycin for the treatment of MRSA.
Note that MRSA infections are mostly acquired in
nosocomial settings. As such the community-acquired
form of the infection has a low incidence, but it is always
a possibility.
10. [C]. Scombroid poisoning has rapid onset, anywhere from
10 minutes to an hour and typically occurs in people
who eat certain fish that have been inadequately
preserved. Scombroid is preformed histamine-like toxin
produced by certain bacteria that grow on fish and it is
resistant to cooking. Fish that are susceptible to the
growth of the bacteria mainly include anchovy,
Australian salmon, bluefish, herring, mackerel, mahimahi, sardine, and tuna.
Note that bacillus cereus (mostly acquired from reheated rice) and Staphylococcus aureus have
incubation of 3-8 hours and they are also acquired from
pre-formed and heat resistant toxins. Also note that
Vibrio parahaemolyticus toxicity is acquired from eating
raw fish. It is responsible for about 50% of foodpoisoning incidents in Japan. This poisoning is
somehow similar to mild forms of cholera and it has an
incubation period of about 15 hours.
11. [A]. Sickle cell disease, cystic fibrosis, vascular
thrombosis and Legg-Calvé-Perthes syndrome are
causes of AVN. Etiology of avascular necrosis of bone is
often aseptic. But frequent infections (osteomyelitis) and
resultant chronic inflammation would also predispose to
avascular necrosis. More commonly, however,
avascular necrosis of the bone is a complication of
corticosteroid
use,
trauma,
systemic
lupus
erythematosus, pancreatitis, alcoholism, gout, and sickle
cell disease. The two organisms that commonly cause
osteomyelitis are salmonella and staph aureus. Children
with sickle cell disease are more prone to recurrent
osteomyelitis due to salmonella because of
splenectomy.
Note that a common cause of AVN in children at the
femoral head is Legg-Calvé-Perthes syndrome (AKA.
Perthes disease) which is an idiopathic degenerative
and ischemic disease related to reduction of blood flow
to the femur head and hip joint most likely as a result of
low flow through the ligamentum teres femoris artery.
56. Northwestern Medical Review, Sample Review Book Series, 2012
Gradual loss of bone mass causes collapse of the hip
joint and deformity of the head of the femur.
12. [D]. Botulinum toxin is produced after germination of the
spores of clostridium botulinum is anaerobic canned
food and it is heat labile.
Note that Staphylococcus aureus, Bacillus cereus and
scombroid food poisonings are acquired from preformed
toxins but they are all heat stable within the household
heating ranges. Salmonella typhi does not cause food
poisoning; it causes enteric fever. Salmonella enteritidis;
the so called salmonella food poisoning, is not due to
preformed toxin but the bug is heat-labile (heatsensitive).
13. [B]. Strep pyogenes is the most common cause of
necrotizing fasciitis
14. [D]. Methicillin causes serious nephrotoxicity. Note that
neither of the two medications is effective against MRSA
and both of them are resistant to beta-lactamases.
15. [D]. All of the cited options may causes sepsis as a
result of skin infection and/or contamination EXCEPT
Clostridium tetani. Note that Klebsiella is quite often
acquired via Foley catheterization.
16. [C]. Next DOC for vancomycin-resistant Staph
organisms is trimethoprim-sulfamethoxazole. Other
choices include Linezolid and Daptomycin.
17. [C]. She has 17-hydrpxylase deficiency. In these
patients conversion of progesterone to subsequent sex
hormones and glucocorticoids is affected. Pathways are
shunted towards mineralocorticoid production and as a
result they are presented with hypokalemia,
hypernatremia and hypertension.
18. [B]. 21-hydroxylase deficiency causes precocious
puberty in males and it is associated with lack of
mineralocorticoids and hypotension. It is contracted with
11-hydroxylase deficiency that causes precocious
puberty and hypertension due to high levels of
deoxycorticosterone.
19. [A]. Addison’s disease is associated with eosinophilia. In
contrast Cushing’s is associated with eosinopenia.
20. [A]. Aminogluthemide inhibits P-450 and in turn inhibits
desmolase activity, and as a result it reduces steroid
output. Additionally it reduces the activity of aromatase
enzyme that converts androgens to estrogens. As such
this drug is quite effective in the management of
estrogen-sensitive breast cancers.
21. [E]. In all cited options with the exception of Waterhouse Friderichsen syndrome the titer of cortisol rises
and inhibits hypothalamic output of corticotropinreleasing hormone (CRH). Water-house Friderichsen
syndrome that is associated with Neisseria meningitidis
bacteremia causes coagulative damages to the adrenal
cortex and acute drop of the adrenocortical hormones.
As a result of decreased cortisol levels and removal of
inhibition over the hypothalmus, CRH is expected to
increase.
Note that Cushing’s disease refers to
hypercortisolism due to adenoma of the anterior pituitary
gland.
22. [C]. The dexamethasone suppression test is for
diagnosis of Cushing's disease. In practice, the test
helps to differentiate the two causes of hypercortisolism
that are both associated with high titers of ACTH and
cortisol. These two causes are ectopic hypercortisolism
(e.g. oat-cell carcinoma of the lung) and pituitary
adenoma. A very high dose of ACTH cannot suppress
the ACTH of ectopic adenomas, but it will suppress the
ACTH output of pituitary adenoma (i.e. Cushing's
disease).
23. [D]. Given her gender, hypertension, and hyperglycemia,
one would expect unilateral adenoma of the adrenal
cortex or adrenal Cushing's as the most probable cause
of this patient's ailment. Note that adrenal adenomas
are often unilateral.
24. [A]. Aldosterone, a product of the zona glomerulosa, is
not controlled by ACTH. After hypophysectomy, all
zones of the adrenals will undergo hypotrophy except
the
zona
glomerulosa.
Aldosterone
and
mineralocorticoids are vital hormones for maintaining
osmotic pressure of the blood within the normal range.
25. [E]. The patient's adrenals are most likely suppressed
as a result of iatrogenic hypercortisolism. Note that
fludrocortisone may also be helpful, but primarily has
mineralocorticoid effects and fewer glucocorticoid
effects compared to prednisone.
26. [B]. Even though all the cited options are related to the
use of corticosteroids, localized candidiasis is among
the earlier findings in this type of situation.
27. [C]. The patient has aortic stenosis (AS) that is causing
a systolic murmur. A common finding in AS is pulsus
tardus et parvus; that is, palpation of the pulse is late
(tardus) relative to contraction of the heart, and also
weak/small (parvus). Almost all AS patients exhibit
systolic murmur. Classic symptoms of aortic stenosis
are angina, syncope, and congestive heart failure.
Note that high pressure in the arm and low in the leg is
characteristic of coarctation of the aorta. Discrepancy of
pressure between the two brachial arteries is
characteristic of Takayasu arteritis.
28. [C]. Semilunar (aortic and pulmonary) stenotic murmurs
and murmurs of atrioventricular valvular (Mitral and
tricuspid) insufficiencies are audible during systole.
29. [A]. The top three causes of paradoxical split are
systemic hypertension, aortic stenosis and left bundle
branch block.
30. [A]. End-systolic ventricular volume increases in AS due
to the fact that less blood is extruded from the left
ventricle.
57. Northwestern Medical Review, Sample Review Book Series, 2012
31. [B]. Among all cited options concentric hypertrophy of
the left ventricle in response to high afterload seems to
have the most drastic effect in increasing the oxygen
demand and leading to ischemia.
Orotic Aciduria causes pyrimidine deficiency and as a
result macrocytic anemia. Liver disease causes
lipidemia. Excess lipids adhere to the membrane of red
cells and as result they appear bigger and macrocytic.
32. [D]. Bicuspid valves have a recessive sex-linked pattern
of inheritance. In females due to presence of a normal
allele on the alternate x-chromosome the phenotype of
bicuspid valves will not be expressed. Additionally, the
bicuspid valves have more tendencies for calcification at
an earlier age. In contrast, in females calcification of the
tricuspid valves as a result of aging predisposes to
aortic stenosis at a very old age.
40. [B]. High levels of 2,3 DPG causes a right shift in the
hemoglobin dissociation curve of oxygen and makes
unloading of oxygen easier at the tissue sides of the
body, and loading of oxygen more difficult in the lungs.
In other words it decreases the affinity of hemoglobin for
oxygen. All other cited conditions will increase the
affinity of hemoglobin for oxygen and make loading
easier and unloading more difficult.
33. [C]. The patient has gigantism, most likely due to
adenoma of the anterior pituitary gland. Tumors of this
nature often press on the optic chiasm and cause
bitemporal heteronymous hemianopia.
41. [D]. Glucose 6 phosphate dehydrogenase deficiency is
by far the most common genetic cause of red blood cell
hemolysis.
34. [A] “Blue Cell” brain tumor of children is
medulloblastoma. It is a highly invasive brain tumor that
arises in the cerebellum. It is the most common
malignant pediatric brain tumor. Histopathologically, this
disease is characterized by sheets of small round blue
cells that are punctuated by frequent mitoses, apoptosis,
and regions of divergent differentiation.
35. [D]. This visual field finding is called central scotoma
and it is not uncommon in patients with multiple
sclerosis. Note that oral contraceptives predispose to
thromboembolism and if a patient presents with
occlusion of both posterior cerebral circulation s/he may
present with bilateral version of this condition. Also note
that occlusion of basilar artery may also theoretically
cause bilateral scotoma, but even if the patients do not
die as a result of occlusion, they may become comatose
and cannot report anything to their doctors.
36. [B]. Right superior quadrantanopia and left inferior
quadrantanopia
37. [E]. Bilateral lesions of parietal and temporal lobe
radiations cause bilateral macular sparing (AKA.
bilateral homonymous macular sparing).
38. [D]. Left superior quadrantanopia ( Pie in the sky!)
39. [A]. The patient has hexokinase deficiency. This enzyme
is a key enzyme of several pathways including
glycolysis, hexose mono-phosphate shunt and glycogen
synthesis. As a result of glycolytic defect, this patient
has hemolytic anemia. Hemolytic conditions cause
normocytic anemias. Kidney diseases also cause
normocytic anemia due to erythropoietin deficiency. On
the other had Thalassemia causes microcytic anemia.
42. [B]. Among the cited options increased parasympathetic
tone is the only condition that raises the titer of insulin
and a result synthesis of PFK II. The latter leads to a
higher production of fructose 2,6 bisphosphate.
Glucagonoma increases the output of glucagon and
drops the glycolytic activities in general, and the level of
fructose 2,6 bisphosphate,
in particular.
Pheochromocytoma increases adrenergic effects and
facilitates release of glucagon. Hexokinase deficiency
drops the levels of fructose 6 phosphate and as a result
formation of both fructose 2,6 bisphosphate and fructose
1,6 bisphosphate.
43. [C]. Phosphofructokinase deficiency is associated with
muscles cramps, red blood cell hemolysis, and
myoglobinuria. The other cited conditions all cause
muscle fatigue and myoglobinuria but they do not
present with hemolysis.
44. [A]. McArdle’s or type V glycogen storage disease is
associated with all of the cited findings in the case
scenario. In this metabolic autosomal recessive disease
muscle glycogen phosphorylase fails to degrade
glycogen. The patient’s cramps are often described as
“electrically silent” cramps because the venous lactate
levels fail to increase upon ischemic exercises. They
often exhibit the so called “second wind” phenomenon
that is characterized by the patient’s better tolerance for
aerobic exercises such as walking or cycling after about
10 minutes of muscular efforts. This is due to a
combination of increased blood flow to muscle and the
ability of the muscle to use alternative sources of
aerobic energy such as fatty acids and proteins. Note
that anaerobic exercise that is accompanied by high
levels of lactate heavily relies on glucose utilization that
is derived from muscle glycogen, especially during the
early phases of exercise.
45. [E]. Chronic myoglobinuria due to any cause may lead
to renal failure. Rhabdomyolysis-induced renal failure
represents up to 15% of all cases of acute renal failure.
Accumulation of myoglobin in the kidney causes either
due to Fenton reaction as a result of presence of free
58. Northwestern Medical Review, Sample Review Book Series, 2012
iron or oxidant injury that damages the cell membrane of
glomerular tissues. Additionally, it is show that presence
of oxidizing agents can also cause renal
vasoconstriction.
46. [C]. Pyruvate kinase accomplishes substrate level
phosphorylation, or in other words the synthesis of ATP
with the conversion into pyruvate of the high energy
compound PEP.
47. [A]. To move pyruvate back from mitochondria to
phosphoenolpyruvate in the cytoplasm during
gluconeogenesis we require two enzymes; pyruvate
carboxylases and phosphoenolpyruvate carboxylases.
Pyruvate carboxylase is a mitochondrial enzyme
whereas phosphoenolpyruvate carboxylase is a
cytoplasmic enzyme.
48. [D]. Hereditary spherocytosis (HS) is an autosomal
dominant condition that is more common in Caucasians.
Likewise most other dominant conditions it is due to
defects in structural proteins. The defective protein in
HS is spectrin that maintains the structural integrity of
the red cell membranes.
49. [B]. The patient has anemia. Among the given options
measurement of mean corpuscular volume (MCV) is the
first step in the diagnosis of the patient as it discerns
among the major three categories of macrocytic,
microcytic and normocytic anemias.
50. [A]. The most dreaded consequence of untreated
anemia is myocardial infarction as a result of failure of
oxygen delivery to the heart muscles.
59. Northwestern Medical Review, Sample Review Book Series, 2012